You are on page 1of 45

Republic of the Philippines which she was issued the corresponding receipt signed by an authorized

SUPREME COURT agent of the respondent insurance corporation. (Rollo, p. 27.) Upon the
Manila filing of said application and the payment of the premium on the policy
applied for, the respondent insurance corporation issued to Carmen O.
FIRST DIVISION Lapuz its Certificate of Insurance No. 128866. (Rollo, p. 28.) The policy
was to be effective for a period of 90 days.
G.R. No. L-34200 September 30, 1982
On May 31, 1969 or during the effectivity of Certificate of Insurance
REGINA L. EDILLON, as assisted by her husband, MARCIAL No. 12886, Carmen O. Lapuz died in a vehicular accident in the North
EDILLON, petitioners-appellants, Diversion Road.
vs.
MANILA BANKERS LIFE INSURANCE CORPORATION and On June 7, 1969, petitioner Regina L. Edillon, a sister of the insured and
the COURT OF FIRST INSTANCE OF RIZAL, BRANCH V, who was the named beneficiary in the policy, filed her claim for the
QUEZON CITY, respondents-appellees. proceeds of the insurance, submitting all the necessary papers and other
requisites with the private respondent. Her claim having been denied,
K.V. Faylona for petitioners-appellants. Regina L. Edillon instituted this action in the Court of First Instance of
Rizal on August 27, 1969.
L. L. Reyes for respondents-appellees.
In resisting the claim of the petitioner, the respondent insurance
VASQUEZ, J.: corporation relies on a provision contained in the Certificate of
Insurance, excluding its liability to pay claims under the policy in behalf
of "persons who are under the age of sixteen (16) years of age or over
The question of law raised in this case that justified a direct appeal from
the age of sixty (60) years ..." It is pointed out that the insured being
a decision of the Court of First Instance Rizal, Branch V, Quezon City,
over sixty (60) years of age when she applied for the insurance
to be taken directly to the Supreme Court is whether or not the
coverage, the policy was null and void, and no risk on the part of the
acceptance by the private respondent insurance corporation of the
respondent insurance corporation had arisen therefrom.
premium and the issuance of the corresponding certificate of insurance
should be deemed a waiver of the exclusionary condition of overage
stated in the said certificate of insurance. The trial court sustained the contention of the private respondent and
dismissed the complaint; ordered the petitioner to pay attorney's fees in
the sum of ONE THOUSAND (P1,000.00) PESOS in favor of the
The material facts are not in dispute. Sometime in April 1969, Carmen
private respondent; and ordered the private respondent to return the sum
O, Lapuz applied with respondent insurance corporation for insurance
of TWENTY (P20.00) PESOS received by way of premium on the
coverage against accident and injuries. She filled up the blank
insurancy policy. It was reasoned out that a policy of insurance being a
application form given to her and filed the same with the respondent
contract of adhesion, it was the duty of the insured to know the terms of
insurance corporation. In the said application form which was dated
the contract he or she is entering into; the insured in this case, upon
April 15, 1969, she gave the date of her birth as July 11, 1904. On the
learning from its terms that she could not have been qualified under the
same date, she paid the sum of P20.00 representing the premium for
INSURANCE 1
conditions stated in said contract, what she should have done is simply ruling that the said deviation from the terms of the policy did not
to ask for a refund of the premium that she paid. It was further argued prevent the claim under the same, this Court stated the following:
by the trial court that the ruling calling for a liberal interpretation of an
insurance contract in favor of the insured and strictly against the insurer We are in agreement with the trial Court that the appellant is barred by
may not be applied in the present case in view of the peculiar facts and waiver (or rather estoppel) to claim violation of the so-called fire
circumstances obtaining therein. hydrants warranty, for the reason that knowing fully an that the number
of hydrants demanded therein never existed from the very beginning,
We REVERSE the judgment of the trial court. The age of the insured the appellant nevertheless issued the policies in question subject to such
Carmen 0. Lapuz was not concealed to the insurance company. Her warranty, and received the corresponding premiums. It would be
application for insurance coverage which was on a printed form perilously close to conniving at fraud upon the insured to allow
furnished by private respondent and which contained very few items of appellant to claim now as void ab initio the policies that it had issued to
information clearly indicated her age of the time of filing the same to be the plaintiff without warning of their fatal defect, of which it was
almost 65 years of age. Despite such information which could hardly be informed, and after it had misled the defendant into believing that the
overlooked in the application form, considering its prominence thereon policies were effective.
and its materiality to the coverage applied for, the respondent insurance
corporation received her payment of premium and issued the The insurance company was aware, even before the policies were
corresponding certificate of insurance without question. The accident issued, that in the premises insured there were only two fire hydrants
which resulted in the death of the insured, a risk covered by the policy, installed by Que Chee Gan and two others nearby, owned by the
occurred on May 31, 1969 or FORTY-FIVE (45) DAYS after the municipality of Tabaco, contrary to the requirements of the warranty in
insurance coverage was applied for. There was sufficient time for the question. Such fact appears from positive testimony for the insured that
private respondent to process the application and to notice that the appellant's agents inspected the premises; and the simple denials of
applicant was over 60 years of age and thereby cancel the policy on that appellant's representative (Jamiczon) can not overcome that proof. That
ground if it was minded to do so. If the private respondent failed to act, such inspection was made it moreover rendered probable by its being a
it is either because it was willing to waive such disqualification; or, prerequisite for the fixing of the discount on the premium to which the
through the negligence or incompetence of its employees for which it insured was entitled, since the discount depended on the number of
has only itself to blame, it simply overlooked such fact. Under the hydrants, and the fire fighting equipment available (See"'Scale of
circumstances, the insurance corporation is already deemed in estoppel. Allowances" to which the policies were expressly made subject). The
It inaction to revoke the policy despite a departure from the law, supported by a long line of cases, is expressed by American
exclusionary condition contained in the said policy constituted a waiver Jurisprudence (Vol. 29, pp. 611-612) to be as follows:
of such condition, as was held in the case of "Que Chee Gan vs. Law
Union Insurance Co., Ltd.,", 98 Phil. 85. This case involved a claim on It is usually held that where the insurer, at the time of the issuance of a
an insurance policy which contained a provision as to the installation of policy of insurance, has knowledge of existing facts which, if insisted
fire hydrants the number of which depended on the height of the on, would invalidate the contract from its very inception, such
external wan perimeter of the bodega that was insured. When it was knowledge constitutes a waiver of conditions in the contract
determined that the bodega should have eleven (11) fire hydrants in the inconsistent with the known facts, and the insurer is stopped thereafter
compound as required by the terms of the policy, instead of only two (2) from asserting the breach of such conditions. The law is charitable
that it had, the claim under the policy was resisted on that ground. In enough to assume, in the absence of any showing to the contrary, that an
INSURANCE 2
insurance company intends to execute a valid contract in return for the Significantly, in the case before Us the Capital Insurance accepted the
premium received; and when the policy contains a condition which promise of Plastic Era to pay the insurance premium within thirty (30)
renders it voidable at its inception, and this result is known to the days from the effective date of policy. By so doing, it has impliedly
insurer, it will be presumed to have intended to waive the conditions and agreed to modify the tenor of the insurance policy and in effect, waived
to execute a binding contract, rather than to have deceived the insured the provision therein that it would only pay for the loss or damage in
into thinking he is insured when in fact he is not, and to have taken is case the same occurs after the payment of the premium. Considering
money without consideration.' (29 Am. Jur., Insurance, section 807, at that the insurance policy is silent as to the mode of payment, Capital
pp. 611-612.) Insurance is deemed to have accepted the promissory note in payment
of the premium. This rendered the policy immediately operative on the
The reason for the rule is not difficult to find. date it was delivered. The view taken in most cases in the United States:

The plain, human justice of this doctrine is perfectly apparent. To allow ... is that although one of conditions of an insurance policy is that "it
a company to accept one's money for a policy of insurance which it then shall not be valid or binding until the first premium is paid", if it is
knows to be void and of no effect, though it knows as it must, that the silent as to the mode of payment, promissory notes received by the
assured believes it to be valid and binding, is so contrary to the dictates company must be deemed to have been accepted in payment of the
of honesty and fair dealing, and so closely related to positive fraud, as to premium. In other words, a requirement for the payment of the first or
be abhorent to fairminded men. It would be to allow the company to initial premium in advance or actual cash may be waived by acceptance
treat the policy as valid long enough to get the premium on it, and leave of a promissory note...
it at liberty to repudiate it the next moment. This cannot be deemed to
be the real intention of the parties. To hold that a literal construction of WHEREFORE, the judgment appealed from is hereby REVERSED and
the policy expressed the true intention of the company would be to SET ASIDE. In lieu thereof, the private respondent insurance
indict it, for fraudulent purposes and designs which we cannot believe it corporation is hereby ordered to pay to the petitioner the sum of TEN
to be guilty of (Wilson vs. Commercial Union Assurance Co., 96 Atl. THOUSAND (P10,000.00) PESOS as proceeds of Insurance Certificate
540, 543544). No. 128866 with interest at the legal rate from May 31, 1969 until fully
paid, the further sum of TWO THOUSAND (P2,000.00) PESOS as and
A similar view was upheld in the case of Capital Insurance & Surety for attorney's fees, and the costs of suit.
Co., Inc. vs. Plastic Era Co., Inc., 65 SCRA 134, which involved a
violation of the provision of the policy requiring the payment of SO ORDERED.
premiums before the insurance shall become effective. The company
issued the policy upon the execution of a promissory note for the Teehankee (Chairman), Makasiar, Plana, Relova and Gutierrez, Jr., JJ.,
payment of the premium. A check given subsequent by the insured as concur.
partial payment of the premium was dishonored for lack of funds.
Despite such deviation from the terms of the policy, the insurer was held Melencio-Herrera, J., took no part.
liable.

INSURANCE 3
failed to act, it was because Manila Bankers was willing to waive such
disqualifications or it simply overlooked such fact. It is therefore
Edillon v. Manila Bankers estopped from disclaiming any liability.
Life Insurance Corp. -
Concealment
117 SCRA 187
Facts:

> In Apr. 1969, Carmen Lapuz applied for insurance with Manila
Bankers. In the application she stated the date of her birth as July 11,
1904 (around 64 yrs old). The policy was thereafter issued. Regina Edillon vs Manila
> Subsequently, in May 1969, Carmen died of a car accident. Her
Banker Life Assurance
sister, as beneficiary claimed the proceeds of the insurance. Corporation
> Manila Bankers refused to pay because the certificate of insurance
contained a provision excluding its liability to pay claims to persons
under 16 or over 60. 117 SCRA 187 Mercantile Law Insurance Law Representation
Collection of Premium Even Though Insured is Disqualified (Age)
Issue:
In April 1969, Carmen Lapuz filled out an application form for
Whether or not the policy is void considering that the insured was over insurance under Manila Banker Life Assurance Corporation. She stated
60 when she applied. that her date of birth was July 11, 1904. Upon payment of the Php 20.00
premium, she was issued the insurance policy in April 1969. In May
Held: 1969, Carmen Lapuz died in a vehicular accident. Regina Edillon, who
was named a beneficiary in the insurance policy sought to collect the
NO. insurance proceeds but Manila Banker denied the claim. Apparently, it
is a rule of the insurance company that they were not to issue insurance
The age of Carmen was not concealed to the insurance company. Her policies to persons who are under the age of sixteen (16) years of age
application form indicated her true age. Despite such information, or over the age of sixty (60) years Note, that Lapuz was already 65
Manila Bankers accepted the premium and issued the policy. It had all years old when she was applying for the insurance policy.
the time to process the application and notice the applicants age. If it
INSURANCE 4
ISSUE: Whether or not Edillon is entitled to the insurance claim as a Have you or any of your family members ever consulted or been treated
beneficiary. for high blood pressure, heart trouble, diabetes, cancer, liver disease,
asthma or peptic ulcer? (If Yes, give details).1
HELD: Yes. Carmen Lapuz did not conceal her true age. Despite this,
the insurance company still received premium from Lapuz and issued The application was approved for a period of one year from March 1,
the corresponding insurance policy to her. When the accident happened, 1988 to March 1, 1989. Accordingly, he was issued Health Care
the insurance policy has been in force for 45 days already and such time Agreement No. P010194. Under the agreement, respondents husband
was already sufficient for Manila Banker to notice the fact that Lapuz is was entitled to avail of hospitalization benefits, whether ordinary or
already over 60 years old and thereby cancel the insurance policy. If emergency, listed therein. He was also entitled to avail of "out-patient
Manila Banker failed to act, it is either because it was willing to waive benefits" such as annual physical examinations, preventive health care
such disqualification; or, through the negligence or incompetence of its and other out-patient services.
employees for which it has only itself to blame, it simply overlooked
such fact. Under the circumstances, Manila Banker is already deemed in Upon the termination of the agreement, the same was extended for
estoppel. another year from March 1, 1989 to March 1, 1990, then from March 1,
1990 to June 1, 1990. The amount of coverage was increased to a
maximum sum of P75,000.00 per disability.2

During the period of his coverage, Ernani suffered a heart attack and
was confined at the Manila Medical Center (MMC) for one month
beginning March 9, 1990. While her husband was in the hospital,
respondent tried to claim the benefits under the health care agreement.
However, petitioner denied her claim saying that the Health Care
Agreement was void. According to petitioner, there was a concealment
G.R. No. 125678 March 18, 2002 regarding Ernanis medical history. Doctors at the MMC allegedly
discovered at the time of Ernanis confinement that he was
PHILAMCARE HEALTH SYSTEMS, INC., petitioner, hypertensive, diabetic and asthmatic, contrary to his answer in the
vs. application form. Thus, respondent paid the hospitalization expenses
COURT OF APPEALS and JULITA TRINOS, respondents. herself, amounting to about P76,000.00.

YNARES-SANTIAGO, J.: After her husband was discharged from the MMC, he was attended by a
physical therapist at home. Later, he was admitted at the Chinese
Ernani Trinos, deceased husband of respondent Julita Trinos, applied for General Hospital. Due to financial difficulties, however, respondent
a health care coverage with petitioner Philamcare Health Systems, Inc. brought her husband home again. In the morning of April 13, 1990,
In the standard application form, he answered no to the following Ernani had fever and was feeling very weak. Respondent was
question: constrained to bring him back to the Chinese General Hospital where he
died on the same day.

INSURANCE 5
On July 24, 1990, respondent instituted with the Regional Trial Court of out that only medical and hospitalization benefits are given under the
Manila, Branch 44, an action for damages against petitioner and its agreement without any indemnification, unlike in an insurance contract
president, Dr. Benito Reverente, which was docketed as Civil Case No. where the insured is indemnified for his loss. Moreover, since Health
90-53795. She asked for reimbursement of her expenses plus moral Care Agreements are only for a period of one year, as compared to
damages and attorneys fees. After trial, the lower court ruled against insurance contracts which last longer,7 petitioner argues that the
petitioners, viz: incontestability clause does not apply, as the same requires an effectivity
period of at least two years. Petitioner further argues that it is not an
WHEREFORE, in view of the forgoing, the Court renders judgment in insurance company, which is governed by the Insurance Commission,
favor of the plaintiff Julita Trinos, ordering: but a Health Maintenance Organization under the authority of the
Department of Health.
1. Defendants to pay and reimburse the medical and hospital coverage
of the late Ernani Trinos in the amount of P76,000.00 plus interest, until Section 2 (1) of the Insurance Code defines a contract of insurance as an
the amount is fully paid to plaintiff who paid the same; agreement whereby one undertakes for a consideration to indemnify
another against loss, damage or liability arising from an unknown or
2. Defendants to pay the reduced amount of moral damages of contingent event. An insurance contract exists where the following
P10,000.00 to plaintiff; elements concur:

3. Defendants to pay the reduced amount of P10,000.00 as exemplary 1. The insured has an insurable interest;
damages to plaintiff;
2. The insured is subject to a risk of loss by the happening of the
4. Defendants to pay attorneys fees of P20,000.00, plus costs of suit. designated peril;

SO ORDERED.3 3. The insurer assumes the risk;

On appeal, the Court of Appeals affirmed the decision of the trial court 4. Such assumption of risk is part of a general scheme to distribute
but deleted all awards for damages and absolved petitioner actual losses among a large group of persons bearing a similar risk; and
Reverente.4 Petitioners motion for reconsideration was
denied.5 Hence, petitioner brought the instant petition for review, raising 5. In consideration of the insurers promise, the insured pays a
the primary argument that a health care agreement is not an insurance premium.8
contract; hence the "incontestability clause" under the Insurance
Code6 does not apply.1wphi1.nt Section 3 of the Insurance Code states that any contingent or unknown
event, whether past or future, which may damnify a person having an
Petitioner argues that the agreement grants "living benefits," such as insurable interest against him, may be insured against. Every person has
medical check-ups and hospitalization which a member may an insurable interest in the life and health of himself. Section 10
immediately enjoy so long as he is alive upon effectivity of the provides:
agreement until its expiration one-year thereafter. Petitioner also points
INSURANCE 6
Every person has an insurable interest in the life and health: Membership Fee according to the mode of payment applied for is
actually paid during the lifetime and good health of proposed Members;
(1) of himself, of his spouse and of his children; that no information acquired by any Representative of PhilamCare shall
be binding upon PhilamCare unless set out in writing in the
(2) of any person on whom he depends wholly or in part for education application;that any physician is, by these presents, expressly authorized
or support, or in whom he has a pecuniary interest; to disclose or give testimony at anytime relative to any information
acquired by him in his professional capacity upon any question affecting
(3) of any person under a legal obligation to him for the payment of the eligibility for health care coverage of the Proposed Members and
money, respecting property or service, of which death or illness might that the acceptance of any Agreement issued on this application shall be
delay or prevent the performance; and a ratification of any correction in or addition to this application as stated
in the space for Home Office Endorsement.11 (Underscoring ours)
(4) of any person upon whose life any estate or interest vested in him
depends. In addition to the above condition, petitioner additionally required the
applicant for authorization to inquire about the applicants medical
history, thus:
In the case at bar, the insurable interest of respondents husband in
obtaining the health care agreement was his own health. The health care
agreement was in the nature of non-life insurance, which is primarily a I hereby authorize any person, organization, or entity that has any record
contract of indemnity.9 Once the member incurs hospital, medical or any or knowledge of my health and/or that of __________ to give to the
other expense arising from sickness, injury or other stipulated PhilamCare Health Systems, Inc. any and all information relative to any
contingent, the health care provider must pay for the same to the extent hospitalization, consultation, treatment or any other medical advice or
agreed upon under the contract. examination. This authorization is in connection with the application for
health care coverage only. A photographic copy of this authorization
shall be as valid as the original.12 (Underscoring ours)
Petitioner argues that respondents husband concealed a material fact
in his application. It appears that in the application for health coverage,
petitioners required respondents husband to sign an express Petitioner cannot rely on the stipulation regarding "Invalidation of
authorization for any person, organization or entity that has any record agreement" which reads:
or knowledge of his health to furnish any and all information relative to
any hospitalization, consultation, treatment or any other medical advice Failure to disclose or misrepresentation of any material information by
or examination.10 Specifically, the Health Care Agreement signed by the member in the application or medical examination, whether
respondents husband states: intentional or unintentional, shall automatically invalidate the
Agreement from the very beginning and liability of Philamcare shall be
We hereby declare and agree that all statement and answers contained limited to return of all Membership Fees paid. An undisclosed or
herein and in any addendum annexed to this application are full, misrepresented information is deemed material if its revelation would
complete and true and bind all parties in interest under the Agreement have resulted in the declination of the applicant by Philamcare or the
herein applied for, that there shall be no contract of health care coverage assessment of a higher Membership Fee for the benefit or benefits
unless and until an Agreement is issued on this application and the full applied for.13

INSURANCE 7
The answer assailed by petitioner was in response to the question contract.17 In this case, no rescission was made. Besides, the cancellation
relating to the medical history of the applicant. This largely depends on of health care agreements as in insurance policies require the
opinion rather than fact, especially coming from respondents husband concurrence of the following conditions:
who was not a medical doctor. Where matters of opinion or judgment
are called for, answers made in good faith and without intent to deceive 1. Prior notice of cancellation to insured;
will not avoid a policy even though they are untrue.14 Thus,
2. Notice must be based on the occurrence after effective date of the
(A)lthough false, a representation of the expectation, intention, belief, policy of one or more of the grounds mentioned;
opinion, or judgment of the insured will not avoid the policy if there is
no actual fraud in inducing the acceptance of the risk, or its acceptance 3. Must be in writing, mailed or delivered to the insured at the address
at a lower rate of premium, and this is likewise the rule although the shown in the policy;
statement is material to the risk, if the statement is obviously of the
foregoing character, since in such case the insurer is not justified in 4. Must state the grounds relied upon provided in Section 64 of the
relying upon such statement, but is obligated to make further inquiry. Insurance Code and upon request of insured, to furnish facts on which
There is a clear distinction between such a case and one in which the cancellation is based.18
insured is fraudulently and intentionally states to be true, as a matter of
expectation or belief, that which he then knows, to be actually untrue, or
None of the above pre-conditions was fulfilled in this case. When the
the impossibility of which is shown by the facts within his knowledge,
terms of insurance contract contain limitations on liability, courts should
since in such case the intent to deceive the insurer is obvious and
construe them in such a way as to preclude the insurer from non-
amounts to actual fraud.15 (Underscoring ours)
compliance with his obligation.19 Being a contract of adhesion, the
terms of an insurance contract are to be construed strictly against the
The fraudulent intent on the part of the insured must be established to party which prepared the contract the insurer.20 By reason of the
warrant rescission of the insurance contract.16 Concealment as a defense exclusive control of the insurance company over the terms and
for the health care provider or insurer to avoid liability is an affirmative phraseology of the insurance contract, ambiguity must be strictly
defense and the duty to establish such defense by satisfactory and interpreted against the insurer and liberally in favor of the insured,
convincing evidence rests upon the provider or insurer. In any case, with especially to avoid forfeiture.21 This is equally applicable to Health Care
or without the authority to investigate, petitioner is liable for claims Agreements. The phraseology used in medical or hospital service
made under the contract. Having assumed a responsibility under the contracts, such as the one at bar, must be liberally construed in favor of
agreement, petitioner is bound to answer the same to the extent agreed the subscriber, and if doubtful or reasonably susceptible of two
upon. In the end, the liability of the health care provider attaches once interpretations the construction conferring coverage is to be adopted,
the member is hospitalized for the disease or injury covered by the and exclusionary clauses of doubtful import should be strictly construed
agreement or whenever he avails of the covered benefits which he has against the provider.22
prepaid.
Anent the incontestability of the membership of respondents husband,
Under Section 27 of the Insurance Code, "a concealment entitles the we quote with approval the following findings of the trial court:
injured party to rescind a contract of insurance." The right to rescind
should be exercised previous to the commencement of an action on the
INSURANCE 8
(U)nder the title Claim procedures of expenses, the defendant The application was approved for 1 year. He was also
Philamcare Health Systems Inc. had twelve months from the date of given hospitalization benefits and out-patient benefits. After the
period expired, he was given an expanded coverage for Php 75,000.
issuance of the Agreement within which to contest the membership of During the period, he suffered from heart attack and was confined at
the patient if he had previous ailment of asthma, and six months from MMC. The wife tried to claim the benefits but the petitioner denied it
the issuance of the agreement if the patient was sick of diabetes or saying that he concealed his medical history by answering no to the
hypertension. The periods having expired, the defense of concealment aforementioned question. She had to pay for the
or misrepresentation no longer lie.23 hospital bills amounting to 76,000. Her husband subsequently
passed away. She filed a case in the trial court for the collection of
the amount plus damages. She was awarded 76,000 for the bills and
Finally, petitioner alleges that respondent was not the legal wife of the 40,000 for damages. The CA affirmed but deleted awards for
deceased member considering that at the time of their marriage, the damages. Hence, this appeal.
deceased was previously married to another woman who was still alive.
The health care agreement is in the nature of a contract of indemnity. Issue: WON a health care agreement is not an insurance contract;
Hence, payment should be made to the party who incurred the expenses. hence the incontestability clause under the Insurance Code does
It is not controverted that respondent paid all the hospital and medical not apply.
expenses. She is therefore entitled to reimbursement. The records
adequately prove the expenses incurred by respondent for the Held: No. Petition dismissed.
deceaseds hospitalization, medication and the professional fees of the
Ratio:
attending physicians.24
Petitioner claimed that it granted benefits only when the insured is
WHEREFORE, in view of the foregoing, the petition is DENIED. The alive during the one-year duration. It contended that there was no
assailed decision of the Court of Appeals dated December 14, 1995 indemnification unlike in insurance contracts. It supported this claim
is AFFIRMED. by saying that it is a health maintenance organization covered by
the DOH and not the Insurance Commission. Lastly, it claimed that
the Incontestability clause didnt apply because two-year and not
SO ORDERED. one-year effectivity periods were required.

Davide, Jr., C.J., Puno, and Kapunan, JJ., concur. Section 2 (1) of the Insurance Code defines a contract of insurance
as an agreement whereby one undertakes for a consideration to
Philamcare v CA G.R. No. 125678. March 18, 2002 indemnify another against loss, damage or liability arising from an
unknown or contingent event.
J. Ynares-Santiago
Section 3 states: every person has an insurable interest in the life
and health:
Facts:

(1) of himself, of his spouse and of his children.


Ernani Trinos applied for a health care coverage with Philam. He
answered no to a question asking if he or his family members were
treated to heart trouble, asthma, diabetes, etc. In this case, the husbands health was the insurable interest. The
health care agreement was in the nature of non-life insurance, which
INSURANCE 9
is primarily a contract of indemnity. The provider must pay for the As to cancellation procedure- Cancellation requires certain
medical expenses resulting from sickness or injury. conditions:

While petitioner contended that the husband concealed materialfact 1. Prior notice of cancellation to insured;
of his sickness, the contract stated that:
2. Notice must be based on the occurrence after effective date of
that any physician is, by these presents, expressly authorized to the policy of one or more of the grounds mentioned;
disclose or give testimony at anytime relative to any information
acquired by him in his professional capacity upon any question 3. Must be in writing, mailed or delivered to the insured at the
affecting the eligibility for health care coverageof the Proposed address shown in the policy;
Members.
4. Must state the grounds relied upon provided in Section 64 of
This meant that the petitioners required him to sign authorization the Insurance Code and upon request of insured, tofurnish facts on
to furnish reports about his medical condition. The contract also which cancellation is based
authorized Philam to inquire directly to his medical history.
None were fulfilled by the provider.
Hence, the contention of concealment isnt valid.
As to incontestability- The trial court said that under the title Claim
They cant also invoke the Invalidation of agreement clause procedures of expenses, the defendant PhilamcareHealth
where failure of the insured to disclose information was agrounds for Systems Inc. had twelve months from the date of issuance of the
revocation simply because the answer assailed by the company was Agreement within which to contest the
the heart condition question based on the insureds opinion. He
wasnt a medical doctor, so he cant accurately gauge his
condition.

Henrick v Fire- in such case the insurer is not justified in relying


upon such statement, but is obligated to make further inquiry.

Fraudulent intent must be proven to rescind the contract. This


was incumbent upon the provider.

Having assumed a responsibility under the agreement, petitioner is


bound to answer the same to the extent agreed upon. In the end,
the liability of the health care provider attaches once the member is
hospitalized for the disease or injury covered by the agreement or
whenever he avails of the covered benefits which he has prepaid.
PHILAMCARE HEALTH SYSTEMS, INC. vs.
Section 27 of the Insurance Code- a concealment entitles the
injured party to rescind a contract of insurance. COURT OF APPEALS

INSURANCE 10
G.R. No. 125678, March 18, 2002 Julita filed an action for damages and reimbursement of her expenses
plus moral damages attorneys fees against Philamcare and its
(YNARES-SANTIAGO, J.) president, Dr. Benito Reverente. The Regional Trial court or Manila
rendered judgment in favor of Julita. On appeal, the decision of the trial
FACTS: court was affirmed but deleted all awards for damages and absolved
petitioner Reverente. Hence, this petition for review raising the primary
Ernani Trinos applied for a health care coverage with Philamcare Health argument that a health care agreement is not an insurance contract;
Systems, Inc. To the question Have you or any of your family hence the incontestability clause under the Insurance Code does
members ever consulted or been treated for high blood pressure, heart not apply.
trouble, diabetes, cancer, liver disease, asthma or peptic ulcer?,
Ernani answered No. Under the agreement, Ernani is entitled to ISSUES:
avail of hospitalization benefits and out-patient benefits. The coverage
was approved for a period of one year from March 1, 1988 to March 1, (1) Whether or not the health care agreement is not an insurance
1989. The agreement was however extended yearly until June 1, 1990 contract
which increased the amount of coverage to a maximum sum of P75,000
per disability. (2) Whether or not there is concealment of material fact made by Ernani

During the period of said coverage, Ernani suffered a heart attack and HELD:
was confined at the Manila Medical Center (MMC) for one month.
While in the hospital, his wife Julita tried to claim the benefits under the (1)YES. Section2 (1)of the Insurance Code defines a contract of
health care agreement. However, the Philamcare denied her claim insurance as an agreement whereby one undertakes for a consideration
alleging that the agreement was void because Ernani concealed his to indemnify another against loss, damage, or liability arising from an
medical history. Doctors at the MMC allegedly discovered at the time of unknown or contingent event.
Ernanis confinement that he was hypertensive, diabetic and
asthmatic, contrary to his answer in the application form. Thus, Julita Section 3 of the Insurance Code states that any contingent or unknown
paid for all the hospitalization expenses. event, whether past or future, which my damnify a person having an
insurable against him, may be insured against. Every person has an
After Ernani was discharged from the MMC, he was attended by a insurable interest in the life and health of himself.
physical therapist at home. Later, he was admitted at the Chinese
General Hospital. Due to financial difficulties, however, respondent Section 10 provides that every person has an insurable interest in the life
brought her husband home again. In the morning of April 13, 1990, and health (1) of himself, of his spouse and of his children.
Ernani had fever and was feeling very weak. Respondent was
constrained to bring him back to the Chinese General Hospital where he The insurable interest of respondents husband in obtaining the health
died on the same day. care agreement was his own health. The health care agreement was in
the nature of non-life insurance, which is primarily a contract of
indemnity. Once the member incurs hospital, medical or any other
INSURANCE 11
expense arising from sickness, injury or other stipulated contingent, the G.R. No. 125678 March 18, 2002
health care provider must pay for the same to the extent agreed upon Lessons Applicable:
Elements (Insurance)
under the contract.
Blood Relationship (Insurance)

(2) NO. The answer assailed by petitioner was in response to the FACTS:
question relating to the medical history of the applicant. This largely
depends on opinion rather than fact, especially coming from Ernani Trinos, deceased husband of Julita Trinos, applied for a health
respondents husband who was not a medical doctor. Where matters of care coverage with Philamcare Health Systems, Inc.
opinion or judgment are called for answers made I good faith and
without intent to deceive will not avoid a policy even though they are He answered the standard application form: Have you or any of your
untrue. family members ever consulted or been treated for high blood
pressure, heart trouble, diabetes, cancer, liver disease, asthma or
peptic ulcer? (If Yes, give details). - NO
The fraudulent intent on the part of the insured must be established to
warrant rescission of the insurance contract. Concealment as a defense the application was approved for a period of one year from March 1,
for the health care provider or insurer to avoid liability is an affirmative 1988 to March 1, 1989. Accordingly, he was issued Health Care
defense and the duty to establish such defense by satisfactory and Agreement No. P010194
convincing evidence rests upon the provider or insurer. In any case, with
or without the authority to investigate, petitioner is liable for claims Under the agreement, respondents husband was entitled to avail
made under the contract. Having assumed a responsibility under the of hospitalization benefits, whether ordinary or emergency, listed
therein. He was also entitled to avail of "out-patient benefits" such
agreement, petitioner is bound to answer to the extent agreed upon. In as annual physical examinations, preventive health care and other
the end, the liability of the health care provider attaches once the out-patient services.
member is hospitalized for the disease or injury covered by the
agreement or wherever he avails of the covered benefits which he has Upon the termination of the agreement, the same was extended for
prepaid. another year from March 1, 1989 to March 1, 1990, then from March
1, 1990 to June 1, 1990. The amount of coverage was increased to a
Being a contract of adhesion, the terms of an insurance contract are to maximum sum of P75,000.00 per disability.
be construed strictly against the party which prepared the contract
During the period of his coverage, Ernani suffered a heart attack and
the insurer. By reason of the exclusive control of the insurance company
was confined at the Manila Medical Center (MMC) for 1 month
over the terms and phraseology of the insurance contract, ambiguity beginning March 9, 1990.
must be strictly interpreted against the insurer and liberally in favor of
the insured, especially to avoid forfeiture. This is equally applicable to While her husband was in the hospital, Julina Trinos tried to claim the
Health Care Agreements. benefits under the health care agreement.

Philamcare Health Systems, Philamcare denied her claim saying that the Health Care Agreement
was void for concealing Ernanis medical history so she paid the
hospitalization expenses of P76,000.00 herself.
Inc. V. CA (2002)
INSURANCE 12
Doctors at the MMC allegedly discovered at the time of Ernanis since Health Care Agreements are only for a period of one year,
confinement that he was hypertensive, diabetic and asthmatic, as compared to insurance contracts which last
contrary to his answer in the application form. longer; incontestability clause does not apply, as the same requires
an effectivity period of at least two years
After being discharged from the MMC, he was attended by a physical
therapist at home. insurance company is governed by the Insurance Commission,
but a Health Maintenance Organization under the authority of the
Later, he was admitted at the Chinese General Hospital. Department of Health

Due to financial difficulties, however, he was brought home again. ISSUE:

April 13, 1990 morning: Ernani had fever and was feeling very weak 1. W/N the health care agreement is a contract of insurance. - YES

He was brought to Chinese General Hospital where he died 2. W/N the spouse being "not" legal wife can claim - YES

July 24, 1990: She brought action for damages against Philamcare
Health Systems Inc. and its president, Dr. Benito Reverente HELD: Petition is DENIED. CA AFFIRMED.

RTC: Philamcare and Dr. Benito Reverent to pay and reimburse P76k 1. YES.
plus interest, moral damages, exemplary damages, attorney's fees
and cost of suit
P.D. 612 Insurance Code
Sec. 2 (1)
CA: affirmed the decision of RTC but deleted all awards for damages
(1) A "contract of insurance" is an agreement whereby one
and absolved Philamcare
undertakes for a consideration to indemnify another against loss,
damage or liability arising from an unknown or contingent event.
Philamcare brought an instant petition for review arguing that: Sec. 3
Sec. 3. Any contingent or unknown event, whether past or
health care agreement is not an insurance contract; hence the future, which may damnify a person having an insurable interest,
"incontestability clause" under the Insurance Code does not or create a liability against him, may be insured against, subject
apply. to the provisions of this chapter.

grants "living benefits," such as medical check-ups and The consent of the husband is not necessary for the validity of
hospitalization which a member may immediately enjoy so long an insurance policy taken out by a married woman on her life or
as he is alive upon effectivity of the agreement until its expiration that of her children.
one-year thereafter
Any minor of the age of eighteen years or more, may,
notwithstanding such minority, contract for life, health and
only medical and hospitalization benefits are given under the accident insurance, with any insurance company duly authorized
agreement without any indemnification, unlike in an insurance to do business in the Philippines, provided the insurance is taken
contract where the insured is indemnified for his loss on his own life and the beneficiary appointed is the minor's

INSURANCE 13
estate or the minor's father, mother, husband, wife, child,
brother or sister. P.D. 612 Insurance Code
The married woman or the minor herein allowed to take out an
insurance policy may exercise all the rights and privileges of an
owner under a policy. Sec. 27

All rights, title and interest in the policy of insurance taken out
by an original owner on the life or health of a minor shall
automatically vest in the minor upon the death of the original Sec. 27. A concealment whether intentional or unintentional
owner, unless otherwise provided for in the policy. entitles the injured party to rescind a contract of insurance.

In the case at bar, the insurable interest of respondent's husband in


obtaining the health care agreement was his own health.
cancellation of health care agreements as in insurance policies
in the nature of non-life insurance, which is primarily a contract of require the concurrence of the following conditions: - none of these
indemnity was made

Once the member incurs hospital, medical or any other expense 1. Prior notice of cancellation to insured;
arising from sickness, injury or other stipulated contingent, the
health care provider must pay for the same to the extent agreed
2. Notice must be based on the occurrence after effective date of the
upon under the contract.
policy of one or more of the grounds mentioned;

The answer in response to the question relating to the medical


3. Must be in writing, mailed or delivered to the insured at the
history of the applicant largely depends on opinion rather than fact,
address shown in the policy;
especially coming from respondent's husband who was not a
medical doctor.
4. Must state the grounds relied upon provided in Section 64 of the
Insurance Code and upon request of insured, to furnish facts on
Where matters of opinion or judgment are called for, answers made
which cancellation is based.
in good faith and without intent to deceive will not avoid a policy
even though they are untrue.
When the terms of insurance contract contain limitations on liability,
courts should construe them in such a way as to preclude the insurer
The fraudulent intent on the part of the insured must be established
from non-compliance with his obligation.
to warrant rescission of the insurance contract.

Being a contract of adhesion, the terms of an insurance contract are


Concealment as a defense for the health care provider or insurer to
to be construed strictly against the party which prepared the
avoid liability is an affirmative defense and the duty to establish
contract - the insurer.
such defense by satisfactory and convincing evidence rests upon the
provider or insurer.

INSURANCE 14
(U)nder the title Claim procedures of expenses, the defendant 379 SCRA 356 Mercantile Law Insurance Law Representation
Philamcare Health Systems Inc. had twelve months from the date of Concealment Rescission of an Insurance Contract Health
issuance of the Agreement within which to contest the membership
of the patient if he had previous ailment of asthma, and six months
Care Agreement is an Insurance Contract
from the issuance of the agreement if the patient was sick of
diabetes or hypertension. The periods having expired, the defense of In 1988, Ernani Trinos applied for a health care insurance under the
concealment or misrepresentation no longer lie. Philamcare Health Systems, Inc. He was asked if he was ever treated for
high blood, heart trouble, diabetes, cancer, liver disease, asthma, or
2. YES. peptic ulcer; he answered no. His application was approved and it was
effective for one year. His coverage was subsequently renewed twice for
P.D. 612 Insurance Code one year each. While the coverage was still in force in 1990, Ernani
Sec. 10 suffered a heart attack for which he was hospitalized. The cost of the
Sec. 10. Every person has an insurable interest in the life and hospitalization amounted to P76,000.00. Julita Trinos, wife of Ernani,
health:
(1) of himself, of his spouse and of his children; filed a claim before Philamcare for the latter to pay the hospitalization
(2) of any person on whom he depends wholly or in part for cost. Philamcare refused to pay as it alleged that Ernani failed to
education or support, or in whom he has a pecuniary interest; disclose the fact that he was diabetic, hypertensive, and asthmatic. Julita
(3) of any person under a legal obligation to him for the payment ended up paying the hospital expenses. Ernani eventually died. In July
of money, respecting property or service, of which death or 1990, Julita sued Philamcare for damages. Philamcare alleged that the
illness might delay or prevent the performance; and
(4) of any person upon whose life any estate or interest vested
health coverage is not an insurance contract; that the concealment made
in him depends. by Ernani voided the agreement.

ISSUE: Whether or not Philamcare can avoid the health coverage


not the legal wife (deceased was previously married to another agreement.
woman who was still alive)
HELD: No. The health coverage agreement (health care agreement)
health care agreement is in the nature of a contract of indemnity. entered upon by Ernani with Philamcare is a non-life insurance contract
and is covered by the Insurance Law. It is primarily a contract of
payment should be made to the party who incurred the expenses indemnity. Once the member incurs hospital, medical or any other
expense arising from sickness, injury or other stipulated contingent, the
Philamcare Health Systems, health care provider must pay for the same to the extent agreed upon
under the contract. There is no concealment on the part of Ernani. He
Inc. vs Court of Appeals answered the question with good faith. He was not a medical doctor
hence his statement in answering the question asked of him when he
was applying is an opinion rather than a fact. Answers made in good
faith will not void the policy.

INSURANCE 15
Further, Philamcare, in believing there was concealment, should have
taken the necessary steps to void the health coverage agreement prior to
the filing of the suit by Julita. Philamcare never gave notice to Julita of G.R. No. L-41014 November 28, 1988
the fact that they are voiding the agreement. Therefore, Philamcare
should pay the expenses paid by Julita. PACIFIC BANKING CORPORATION, petitioner,
vs.
COURT OF APPEALS and ORIENTAL ASSURANCE
CORPORATION, respondents.

PARAS, J.:

This is a petition for review on certiorari of the decision of respondent


Court of Appeals * in CA-G.R. No. 41735-R, entitled "Pacific Banking
Corporation vs. Oriental Assurance Corporation", which set aside the
decision of the Court of First Instance (CFI) of Manila, ** which had in
turn granted the complaint for a sum of money in Civil Case No. 56889.

As gathered from the records, the undisputed facts of this case are as
follows:

On October 21,1963, Fire Policy No. F-3770 (Exhibit "A"), an open


policy, was issued to the Paramount Shirt Manufacturing Co.
(hereinafter referred to as the insured, for brevity), by which private
respondent Oriental Assurance Corporation bound itself to indemnify
the insured for any loss or damage, not exceeding P61,000.00, caused
by fire to its property consisting of stocks, materials and supplies usual
to a shirt factory, including furniture, fixtures, machinery and equipment
while contained in the ground, second and third floors of the building
situated at number 256 Jaboneros St., San Nicolas, Manila, for a period
of one year commencing from that date to October 21, 1964.

The insured was at the time of the issuance of the policy and is up to
this time, a debtor of petitioner in the amount of not less than Eight
Hundred Thousand Pesos (P800,000.00) and the goods described in the
policy were held in trust by the insured for the petitioner under thrust
receipts (Record on Appeal, p. 4).
INSURANCE 16
Said policy was duly endorsed to petitioner as mortgagee/ trustor of the On May 25, 1964, private respondent raised the following defenses in
properties insured, with the knowledge and consent of private its answer to wit: (a) lack of formal claim by insured over the loss and
respondent to the effect that "loss if any under this policy is payable to (b) premature filing of the suit as neither plaintiff nor insured had
the Pacific Banking Corporation". submitted any proof of loss on the basis of which defendant would
determine its liability and the amount thereof, either to the private
On January 4, 1964, while the aforesaid policy was in full force and respondent or its ad . adjuster H.H. Bayne Adjustment Co., both in
effect, a fire broke out on the subject premises destroying the goods violation of Policy Condition No.11 (Record on Appeal, pp. 37-38).
contained in its ground and second floors (Record on Appeal, p.5)
At the trial, petitioner presented in evidence Exhibit "H", which is a
On January 24, 1964, counsel for the petitioner sent a letter of demand communication dated December 22, 1965 of the insurance adjuster,
to private respondent for indemnity due to the loss of property by fire H.H. Bayne Adjustment Co. to Asian Surety Insurance Co., Inc.,
under the endorsement of said policy (Brief for Plaintiff-Appellee, pp. revealing undeclared co-insurances with the following: P30,000.00 with
16-17). Wellington Insurance; P25,000. 00 with Empire Surety and
P250,000.00 with Asian Surety; undertaken by insured Paramount on
On January 28, 1964, private respondent informed counsel for the the same property covered by its policy with private respondent whereas
petitioner that it was not yet ready to accede to the latter's demand as the the only co-insurances declared in the subject policy are those
former is awaiting the final report of the insurance adjuster, H.H. Bayne of P30,000.00 with Malayan P50,000.00 with South
Adjustment Company (Brief for Plaintiff-Appellee, pp. 17-18). Sea and P25.000.00 with Victory (Brief for the Defendant pp. 13-14).

On March 25, 1964, the said insurance adjuster notified counsel for the It will be noted that the defense of fraud and/or violation of Condition
petitioner that the insured under the policy had not filed any claim with No. 3 in the Policy, in the form of non-declaration of co-insurances
it, nor submitted proof of loss which is a clear violation of Policy which was not pleaded in the answer was also not pleaded in the Motion
Condition No.11, and for which reason, determination of the liability of to Dismiss.
private respondent could not be had (Supra, pp. 19-20).
At any rate, on June 30, 1967, the trial court denied private respondent's
On April 24, 1964, petitioner's counsel replied to aforesaid letter asking motion on the ground that the defense of lack of proof of loss or defects
the insurance adjuster to verify from the records of the Bureau of therein was raised for the first time after the commencement of the suit
Customs the entries of merchandise taken into the customs bonded and that it must be deemed to have waived the requirement of proof of
warehouse razed by fire as a reliable proof of loss (Supra, pp. 21-22). loss (Sections 83 and 84, Insurance Act; Record on Appeal, p. 61).
For failure of the insurance company to pay the loss as demanded,
petitioner (plaintiff therein) on April 28, 1 964, filed in the court a On September 9, 1967, the case was considered submitted for decision
quo an action for a sum of money against the private respondent, from which order private respondent filed a motion for reconsideration
Oriental Assurance Corporation, in the principal sum of P61,000.00 to set the case or further reception of private respondent's additional
issued in favor of Paramount Shirt Manufacturing Co. (Record on evidence, "in order to prove that 'insured has committed a violation of
Appeal, pp. 1-36). condition No. 3 of the policy in relation to the other Insurance Clause.' "
(Record on Appeal, pp. 61-69).

INSURANCE 17
On September 30,1967, the case was set for the continuation of the (a) The respondent Court did not consider the legal presumption against
hearing for the reception merely of the testimony of Alejandro Tan the existence of fraud, which should be established with such quantum
Gatue, Manager of the Adjustment Co., over the vehement opposition of of proof as is required for any crime.
the petitioner (Record on Appeal, p. 129).
(b) The record of the case is bereft of proof of such fraud.
On April 18, 1 968, the trial court rendered a decision adjudging private
respondent liable to the petitioner under the said contract of insurance, (c) The private respondent insurer did not even plead or in anywise raise
the dispositive portion of which reads: fraud as a defense in its answer or motion to dismiss and, therefore, it
should have been considered waived.
WHEREFORE, judgment is hereby rendered ordering the defendant to
pay the plaintiff P61,000.00, with interest at the rate of 8% per annum (d) The total amount of insurance procured by the insured from the
from January 4, 1964, to April 28, 1964, and 12% from April 29, 1964, different companies amounted to hardly onehalf () of the value of the
until the amount is fully paid, P6,100.00, as attorney's fees, and the goods insured.
costs.
II
SO ORDERED. (Record on Appeal, pp. 140-141)
RESPONDENT COURT ERRED IN NOT HOLDING THAT
On appeal, the Court of Appeals reversed the decision of the trial court CONSIDERING THE VOTING ON THE PARTICULAR QUESTION
(Decision promulgated on April 23, 1975, Rollo, pp. 21-33). OF FRAUD, THE FINDING OF THE TRIAL COURT THEREON
SHOULD BE CONSIDERED AFFIRMED.
Petitioner filed a motion for reconsideration of the said decision of the
respondent Court of Appeals, but this was denied on July 3,1975 for III
lack of merit (Rollo, pp. 54-67), resulting in this petition with the
following assigned errors; THE CONCURRING OPINION OF MR. JUSTICE CHANCO IS
LEGALLY ERRONEOUS IN HOLDING THAT THE ACTION WAS
I PREMATURELY BROUGHT BECAUSE THE REQUIRED CLAIM
UNDER THE INSURANCE LAW HAS NOT BEEN FILED,
RESPONDENT COURT OF APPEALS COMMITTED A GRAVE NOTWITHSTANDING THE LETTER, (EXHIBIT "C") OF
ERROR OF LAW IN CONCLUDING FRAUD FROM THE BARE PETITIONER-APPELLANT'S LAWYER WHICH IS A
FACT THAT THE INSURED PARAMOUNT PROCURED SUBSTANTIAL COMPLIANCE OF THE LEGAL REQUIREMENTS
ADDITIONAL INSURANCES OTHER THAN THOSE STATED IN AND NOT HOLDING THAT PRIVATE RESPONDENT INSURER
THE POLICY IN SPITE OF THE EXISTENCE OF CONTRARY HAD ALREADY WAIVED THE SUPPOSED DEFECTS IN THE
PRESUMPTIONS AND ADMITTED FACT AND CIRCUMSTANCES CLAIM FILED BY PETITIONER-APPELLANT FOR ITS FAILURE
WHICH NEGATE THE CORRECTNESS OF SAID CONCLUSION. TO CALL THE ATTENTION OF THE LAYER TO SUCH ALLEGED
DEFECTS AND FOR ENDORSING THE CLAIM TO ITS ADJUSTER
FOR PROCESSING.
INSURANCE 18
IV such contract. Hence, the insured was guilty of clear fraud (Rollo, p.
25).
RESPONDENT COURT OF APPEALS COMMITTED A GRAVE
ERROR OF LAW IN NOT INTERPRETING THE PROVISIONS OF Petitioner's contention that the allegation of fraud is but a mere
THE POLICY LIBERALLY IN FAVOR OF THE HEREIN inference or suspicion is untenable. In fact, concrete evidence of fraud
PETITIONER-APPELLANT, WHO IS NOT THE INSURED BUT or false declaration by the insured was furnished by the petitioner itself
ONLY THE ASSIGNEE/MORTGAGEE OF THE PROPERTY when the facts alleged in the policy under clauses "Co-Insurances
INSURED. Declared" and "Other Insurance Clause" are materially different from
the actual number of co-insurances taken over the subject property.
V Consequently, "the whole foundation of the contract fails, the risk does
not attach and the policy never becomes a contract between the parties.
RESPONDENT COURT OF APPEALS COMMITTED A GRAVE Representations of facts are the foundation of the contract and if the
ERROR OF LAW IN DISMISSING THE CASE AND IN NOT foundation does not exist, the superstructure does not arise. Falsehood
AFFIRMING THE APPEALED DECISION OF THE TRIAL COURT. in such representations is not shown to vary or add to the contract, or to
(Brief for Petitioners, pp. 1-3) terminate a contract which has once been made, but to show that no
contract has ever existed (Tolentino, Commercial Laws of the
The crux of the controversy centers on two points: (a) unrevealed co- Philippines, p. 991, Vol. II, 8th Ed.) A void or inexistent contract is one
insurances which violated policy conditions No. 3 and (b) failure of the which has no force and effect from the very beginning, as if it had never
insured to file the required proof of loss prior to court action. Policy been entered into, and which cannot be validated either by time or by
Condition No. 3 explicitly provides: ratification Tongoy v. C.A., 123 SCRA 99 [1983]; Avila v. C.A. 145
SCRA [1986]).
3. The Insured shall give notice to the Company of any insurance
already effected, or which may subsequently be effected, covering any As the insurance policy against fire expressly required that notice
of the property hereby insured, and unless such notice be given and the should be given by the insured of other insurance upon the same
particulars of such insurance or insurances be stated in or endorsed on property, the total absence of such notice nullifies the policy (Sta. Ana v.
this Policy by or on behalf of the Company before the occurrence of any Commercial Union Assurance Co., 55 Phil. 333 [1930]; Union
loss or damage, all benefit under this policy shall be forfeited. (Record Manufacturing Co., Inc. vs. Philippine Guaranty Co., Inc., 47 SCRA
on Appeal, p. 12) 276 [1972]; Pioneer Ins. & Surety Corp., v. Yap, 61 SCRA 432 [1974]).

It is not disputed that the insured failed to reveal before the loss three The argument that notice of co-insurances may be made orally is
other insurances. As found by the Court of Appeals, by reason of said preposterous and negates policy condition No. 20 which requires every
unrevealed insurances, the insured had been guilty of a false notice and other communications to the insurer to be written or printed.
declaration; a clear misrepresentation and a vital one because where the
insured had been asked to reveal but did not, that was deception. Petitioner points out that Condition No. 3 in the policy in relation to the
Otherwise stated, had the insurer known that there were many co- "other insurance clause" supposedly to have been violated, cannot
insurances, it could have hesitated or plainly desisted from entering into certainly defeat the right of the petitioner to recover the insurance as

INSURANCE 19
mortgagee/assignee. Particularly referring to the mortgage clause of the Petitioner further stressed that fraud which was not pleaded as a defense
policy, petitioner argues that considering the purpose for which the in private respondent's answer or motion to dismiss, should be deemed
endorsement or assignment was made, that is, to protect the to have been waived.
mortgagee/assignee against any untoward act or omission of the insured,
it would be absurd to hold that petitioner is barred from recovering the It will be noted that the fact of fraud was tried by express or at least
insurance on account of the alleged violation committed by the insured implied consent of the parties. Petitioner did not only object to the
(Rollo, Brief for the petitioner, pp, 33-35). introduction of evidence but on the contrary, presented the very
evidence that proved its existence.
It is obvious that petitioner has missed all together the import of subject
mortgage clause which specifically provides: Be that as it may, it is established that the Supreme Court has ample
authority to give beyond the pleadings where in the interest of justice
Mortgage Clause and the promotion of public policy, there is a need to make its own
finding to support its conclusion. Otherwise stated, the Court can
Loss, if any, under this policy, shall be payable to the PACIFIC consider a fact which surfaced only after trial proper (Maharlika
BANKING CORPORATION Manila mortgagee/trustor as its interest Publishing Corp. v. Tagle, 142 SCRA 561 [1986]).
may appear, it being hereby understood and agreed that this insurance as
to the interest of the mortgagee/trustor only herein, shall not be Generally, the cause of action on the policy accrues when the loss
invalidated by any act or neglectexcept fraud or misrepresentation, or occurs, But when the policy provides that no action shall be brought
arsonof the mortgagor or owner/trustee of the property insured; unless the claim is first presented extrajudicially in the manner provided
provided, that in case the mortgagor or owner/ trustee neglects or in the policy, the cause of action will accrue from the time the insurer
refuses to pay any premium, the mortgagee/ trustor shall, on demand finally rejects the claim for payment (Eagle Star Insurance v. Chia Yu,
pay the same. (Rollo, p. 26) 55 Phil 701 [1955]).

The paragraph clearly states the exceptions to the general rule that In the case at bar, policy condition No. 11 specifically provides that the
insurance as to the interest of the mortgagee, cannot be invalidated; insured shall on the happening of any loss or damage give notice to the
namely: fraud, or misrepresentation or arson. As correctly found by the company and shall within fifteen (15) days after such loss or damage
Court of Appeals, concealment of the aforecited deliver to the private respondent (a) a claim in writing giving particular
co-insurances can easily be fraud, or in the very least, misrepresentation account as to the articles or goods destroyed and the amount of the loss
(Rollo, p. 27). or damage and (b) particulars of all other insurances, if any. Likewise,
insured was required "at his own expense to produce, procure and give
Undoubtedly, it is but fair and just that where the insured who is to the company all such further particulars, plans, specifications, books,
primarily entitled to receive the proceeds of the policy has by its fraud vouchers, invoices, duplicates or copies thereof, documents, proofs and
and/or misrepresentation, forfeited said right, with more reason information with respect to the claim". (Record on Appeal, pp. 18-20).
petitioner which is merely claiming as indorsee of said insured, cannot
be entitled to such proceeds. The evidence adduced shows that twenty-four (24) days after the fire,
petitioner merely wrote letters to private respondent to serve as a notice
INSURANCE 20
of loss, thereafter, the former did not furnish the latter whatever v. Alonzo, III SCRA 341 [1982]; Gonzales v. CA, 124 SCRA 630
pertinent documents were necessary to prove and estimate its loss. [1983]; GSIS v. CA, 145 SCRA 311 [1986]; Herrera v. Petrophil Corp.,
Instead, petitioner shifted upon private respondent the burden of fishing 146 SCRA 385 [1986]).
out the necessary information to ascertain the particular account of the
articles destroyed by fire as well as the amount of loss. It is noteworthy Contracts of insurance are contracts of indemnity upon the terms and
that private respondent and its adjuster notified petitioner that insured conditions specified in the policy. The parties have a right to impose
had not yet filed a written claim nor submitted the supporting such reasonable conditions at the time of the making of the contract as
documents in compliance with the requirements set forth in the policy. they may deem wise and necessary. The agreement has the force of law
Despite the notice, the latter remained unheedful. Since the required between the parties. The terms of the policy constitute the measure of
claim by insured, together with the preliminary submittal of relevant the insurer's liability, and in order to recover, the insured must show
documents had not been complied with, it follows that private himself within those terms. The compliance of the insured with the
respondent could not be deemed to have finally rejected petitioner's terms of the policy is a condition precedent to the light of recovery
claim and therefore the latter's cause of action had not yet arisen. (Stokes v. Malayan Insurance Co., Inc., 127 SCRA 766 [1984]).
Compliance with condition No. 11 is a requirement sine qua non to the
right to maintain an action as prior thereto no violation of petitioner's It appearing that insured has violated or failed to perform the conditions
right can be attributable to private respondent. This is so, as before such under No. 3 and 11 of the contract, and such violation or want of
final rejection, there was no real necessity for bringing suit. Petitioner performance has not been waived by the insurer, the insured cannot
should have endeavored to file the formal claim and procure all the recover, much less the herein petitioner. Courts are not permitted to
documents, papers, inventory needed by private respondent or its make contracts for the parties; the function and duty of the courts is
adjuster to ascertain the amount of loss and after compliance await the simply to enforce and carry out the contracts actually made (Young v.
final rejection of its claim. Indeed, the law does not encourage Midland Textile Ins. Co., 30 Phil. 617 [1915]; Union Manufacturing Co.
unnecessary litigation (Eagle Star Insurance Co., Ltd., et al. v. Chia Yu, Inc. v. Phil. Guaranty Co. Inc., p. 276 supra).
p. 701, supra).
Finally, the established rule in this jurisdiction that findings of fact of
Verily, petitioner prematurely filed Civil Case No. 56889 and dismissal the Court of Appeals when supported by substantial evidence, are not
thereof was warranted under the circumstances. While it is a cardinal reviewable on appeal by certiorari, deserves reiteration. Said findings of
principle of insurance law that a policy or contract of insurance is to be the appellate court are final and cannot be disturbed by the Supreme
construed liberally in favor of the insured and strictly as against the Court except in certain cases Lereos v. CA, 117 SCRA 395 [1985];
insurer company (Eagle Star Insurance Co., Ltd., et al. v. Chia Yu, p. Dalida v. CA, 117 SCRA 480 [1982] Director of Lands v. CA, 117
702, supra; Taurus Taxi Co., Inc. v. The Capital Ins. & Surety Co., Inc., SCRA 346 [1982]; Montesa v. CA, 117 SCRA 770 [1982]; Sacay v.
24 SCRA 458 [1968]; National Power Corp. v. CA, 145 SCRA 533 Sandiganbayan, 142 SCRA 609 [1986]; Guita v. CA, 139 SCRA 576
[1986]), yet, contracts of insurance, like other contracts, are to be [1985]; Manlapaz v. CA, 147 SCRA 238-239 [1987]).
construed according to the sense and meaning of the terms which the
parties themselves have used. If such terms are clear and unambiguous, PREMISES CONSIDERED, the petition is DISMISSED for lack of
they must be taken and understood in their plain, ordinary and popular merit, and the decision appealed from is AFFIRMED. No costs.
sense (Young v. Midland Textile Ins. Co., 30 Phil. 617 [1919]; Union
Manufacturing Co., Inc. v. Phil. Guaranty Co., Inc., p. 277 supra; Pichel
INSURANCE 21
SO ORDERED. respondent to the effect that "loss if any under this policy
is payable to the Pacific Banking Corporation".
Melencio-Herrera, (Chairman), Padilla, Sarmiento and Regalado, JJ.,
A fire broke out on the premises destroying the goods contained in
concur. the building.

The bank sent a letter of demand to Oriental for indemnity.

The company wasnt ready to give since it was awaiting the


adjusters report.

The company then made an excuse that the insured had not filed
any claim with it, nor submitted proof of loss which is a clear
violation of Policy Condition No.11, as a result, determination of the
liability of private respondent could not be made.

Pacific Banking filed in the trial court an action for a sum of money
for P61,000.00 against Oriental Assurance.

At the trial, petitioner presented communications of the


insurance adjuster to Asian Surety revealing undeclared co-
insurances with the following: P30,000 with Wellington Insurance;
P25,000 with Empire Surety and P250,000 with Asian Surety
Pacific v CA G.R. No. L-41014 November 28, 1988 undertaken by insured Paramount on the same property covered by
J. Paras its policy with Oriental whereas the only co-insurances declared in
the subject policy are those of P30,000.00 with Malayan P50,000.00
with South Sea and P25.000.00 with Victory.
Facts:

The defense of fraud, in the form of non-declaration of co-insurances


An open fire insurance policy, was issued to Paramount Shirt
which was not pleaded in the answer, was also not pleaded in the
Manufacturing by Oriental Assurance Corporation to indemnify
Motion to Dismiss.
P61,000.00, caused by fire to the factorys stocks, materials and
supplies.
The trial court denied the respondents motion. Oriental filed
another motion to include additional evidence of the co-insurance
The insured was a debtor of Pacific Banking in the amount of
which could amount to fraud.
(P800,000.00) and the goods described in the policy were held in
trust by the insured for Pacific Banking under trust receipts.
The trial court still made Oriental liable for P 61,000. The CA
reversed the trial court decision. Pacific Banking filed a motion for
The policy was endorsed to Pacific Banking as mortgagee/ trustor of
reconsideration of the said decision of the respondent Court
the properties insured, with the knowledge and consent of private
of Appeals, but this was denied for lack of merit.

INSURANCE 22
Issues: purpose for which the endorsement or assignment was made was to
protect the mortgagee/assignee against any untoward act or
1. WON unrevealed co-insurances Violated policy conditions No. 3 omission of the insured. It would be absurd to hold that petitioner is
barred from recovering the insurance on account of the alleged
violation committed by the insured.
2. WON the insured failed to file the required proof of loss prior to
court action.
It is obvious that petitioner has missed all together the import
of subject mortgage clause which specifically provides:
Held: Yes. Petition dismissed.
Loss, if any, under this policy, shall be payable to the
Ratio: PACIFIC BANKING CORPORATION Manila mortgagee/trustor as its
interest may appear, it being hereby understood and agreed that
1. Policy Condition No. 3 explicitly provides: this insurance as to the interest of the mortgagee/trustor only
herein, shall not be invalidated by any act or neglectexcept fraud
3. The Insured shall give notice to the Company of any insurance or misrepresentation, or arsonof the mortgagor or owner/trustee of
already effected, or which may subsequently be effected, covering the property insured; provided, that in case the mortgagor or owner/
any of the property hereby insured, and unless such notice be given trustee neglects or refuses to pay any premium, the mortgagee/
and the particulars of such insurance or insurances be stated in or trustor shall, on demand pay the same.
endorsed on this Policy by or on behalf of the Company before the
occurrence of any loss or damage, all benefit under this policy shall The paragraph clearly states the exceptions to the general rule that
be forfeited. insurance as to the interest of the mortgagee, cannot be invalidated;
namely: fraud, or misrepresentation or arson. Concealment of the
The insured failed to reveal before the loss three other insurances. aforecited co-insurances can easily be fraud, or in the very least,
Had the insurer known that there were many co-insurances, it could misrepresentation.
have hesitated or plainly desisted from entering into such contract.
Hence, the insured was guilty of clear fraud. Undoubtedly, it is but fair and just that where the insured who is
primarily entitled to receive the proceeds of the policy has by its
Concrete evidence of fraud or false declaration by the insured was fraud and/or misrepresentation, forfeited said right.
furnished by the petitioner itself when the facts alleged in the policy
under clauses "Co-Insurances Declared" and "Other Insurance Petitioner further stressed that fraud which was not pleaded as a
Clause" are materially different from the actual number of co- defense in private respondent's answer or motion to dismiss, should
insurances taken over the subject property. be deemed to have been waived. It will be noted that the fact of
fraud was tried by express or at least implied consent of the parties.
As the insurance policy against fire expressly required that notice Petitioner did not only object to the introduction of evidence but on
should be given by the insured of other insurance upon the same the contrary, presented the very evidence that proved its existence.
property, the total absence of such notice nullifies the policy.
2. Generally, the cause of action on the policy accrues when the loss
Petitioner points out that Condition No. 3 in the policy in relation to occurs, But when the policy provides that no action shall be brought
the "other insurance clause" supposedly to have been violated, unless the claim is first presented extrajudicially in the manner
cannot certainly defeat the right of the petitioner to recover the provided in the policy, the cause of action will accrue from the time
insurance as mortgagee/assignee. Hence, they claimed that the the insurer finally rejects the claim for payment

INSURANCE 23
In the case at bar, policy condition No. 11 specifically provides that GUTIERREZ, JR., J.:
the insured shall on the happening of any loss or damage give notice
to the company and shall within fifteen (15) days after such loss or
damage deliver to the private respondent (a) a claim in writing This is a petition for review on certiorari of the Court of Appeals'
giving particular account as to the articles or goods destroyed and decision affirming the decision of the Insurance Commissioner which
the amount of the loss or damage and (b) particulars of all other dismissed the petitioners' complaint against respondent Philippine
insurances, if any. American Life Insurance Company for the recovery of the proceeds
from their late father's policy. The facts of the case as found by the
Twenty-four days after the fire did petitioner merely wrote letters to Court of Appeals are:
private respondent to serve as a notice of loss. It didnt even
furnish other documents. Instead, petitioner shifted upon private
respondent the burden of fishing out the necessary information to Petitioners appeal from the Decision of the Insurance Commissioner
ascertain the particular account of the articles destroyed by fire as dismissing herein petitioners' complaint against respondent Philippine
well as the amount of loss. Since the required claim by insured, American Life Insurance Company for the recovery of the proceeds of
together with the preliminary submittal of relevant documents had Policy No. 1082467 in the amount of P 80,000.00.
not been complied with, it follows that private respondent could not
be deemed to have finally rejected petitioner's claim and therefore
there was no cause of action. On September 23,1973, Tan Lee Siong, father of herein petitioners,
applied for life insurance in the amount of P 80,000.00 with respondent
It appearing that insured has violated or failed to perform the company. Said application was approved and Policy No. 1082467 was
conditions under No. 3 and 11 of the contract, and such violation or issued effective November 6,1973, with petitioners the beneficiaries
want of performance has not been waived by the insurer, the insured thereof (Exhibit A).
cannot recover, much less the herein petitioner.

On April 26,1975, Tan Lee Siong died of hepatoma (Exhibit B).


Petitioners then filed with respondent company their claim for the
proceeds of the life insurance policy. However, in a letter dated
September 11, 1975, respondent company denied petitioners' claim and
rescinded the policy by reason of the alleged misrepresentation and
G.R. No. 48049 June 29, 1989
concealment of material facts made by the deceased Tan Lee Siong in
his application for insurance (Exhibit 3). The premiums paid on the
EMILIO TAN, JUANITO TAN, ALBERTO TAN and ARTURO policy were thereupon refunded .
TAN, petitioners,
vs.
Alleging that respondent company's refusal to pay them the proceeds of
THE COURT OF APPEALS and THE PHILIPPINE AMERICAN
the policy was unjustified and unreasonable, petitioners filed on
LIFE INSURANCE COMPANY, respondents.
November 27, 1975, a complaint against the former with the Office of
the Insurance Commissioner, docketed as I.C. Case No. 218.
O.F. Santos & P.C. Nolasco for petitioners.

Ferry, De la Rosa and Associates for private respondent.

INSURANCE 24
After hearing the evidence of both parties, the Insurance Commissioner Section 48. Whenever a right to rescind a contract of insurance is given
rendered judgment on August 9, 1977, dismissing petitioners' complaint. to the insurer by any provision of this chapter, such right must be
(Rollo, pp. 91-92) exercised previous to the commencement of an action on the contract.

The Court of Appeals dismissed ' the petitioners' appeal from the After a policy of life insurance made payable on the death of the insured
Insurance Commissioner's decision for lack of merit shall have been in force during the lifetime of the insured for a period of
two years from the date of its issue or of its last reinstatement, the
Hence, this petition. insurer cannot prove that the policy is void ab initio or is rescindable by
reason of the fraudulent concealment or misrepresentation of the insured
The petitioners raise the following issues in their assignment of errors, or his agent.
to wit:
According to the petitioners, the Insurance Law was amended and the
A. The conclusion in law of respondent Court that respondent insurer second paragraph of Section 48 added to prevent the insurance company
has the right to rescind the policy contract when insured is already dead from exercising a right to rescind after the death of the insured.
is not in accordance with existing law and applicable jurisprudence.
The so-called "incontestability clause" precludes the insurer from
B. The conclusion in law of respondent Court that respondent insurer raising the defenses of false representations or concealment of material
may be allowed to avoid the policy on grounds of concealment by the facts insofar as health and previous diseases are concerned if the
deceased assured, is contrary to the provisions of the policy contract insurance has been in force for at least two years during the insured's
itself, as well as, of applicable legal provisions and established lifetime. The phrase "during the lifetime" found in Section 48 simply
jurisprudence. means that the policy is no longer considered in force after the insured
has died. The key phrase in the second paragraph of Section 48 is "for a
C. The inference of respondent Court that respondent insurer was misled period of two years."
in issuing the policy are manifestly mistaken and contrary to admitted
evidence. (Rollo, p. 7) As noted by the Court of Appeals, to wit:

The petitioners contend that the respondent company no longer had the The policy was issued on November 6,1973 and the insured died on
right to rescind the contract of insurance as rescission must allegedly be April 26,1975. The policy was thus in force for a period of only one
done during the lifetime of the insured within two years and prior to the year and five months. Considering that the insured died before the two-
commencement of action. year period had lapsed, respondent company is not, therefore, barred
from proving that the policy is void ab initio by reason of the insured's
The contention is without merit. fraudulent concealment or misrepresentation. Moreover, respondent
company rescinded the contract of insurance and refunded the
premiums paid on September 11, 1975, previous to the commencement
The pertinent section in the Insurance Code provides:
of this action on November 27,1975. (Rollo, pp. 99-100)

INSURANCE 25
xxx xxx xxx We call attention to what this Honorable Court said in Insular Life v.
Feliciano, et al., 73 Phil. 201; at page 205:
The petitioners contend that there could have been no concealment or
misrepresentation by their late father because Tan Lee Siong did not It is of common knowledge that the selling of insurance today is
have to buy insurance. He was only pressured by insistent salesmen to subjected to the whirlwind pressureof modern salesmanship.
do so. The petitioners state:
Insurance companies send detailed instructions to their agents to solicit
Here then is a case of an assured whose application was submitted and procure applications.
because of repeated visits and solicitations by the insurer's agent.
Assured did not knock at the door of the insurer to buy insurance. He These agents are to be found all over the length and breadth of the land.
was the object of solicitations and visits. They are stimulated to more active efforts by contests and by the keen
competition offered by the other rival insurance companies.
Assured was a man of means. He could have obtained a bigger
insurance, not just P 80,000.00. If his purpose were to misrepresent and They supply all the information, prepare and answer the applications,
to conceal his ailments in anticipation of death during the two-year submit the applications to their companies, conclude the transactions,
period, he certainly could have gotten a bigger insurance. He did not. and otherwise smooth out all difficulties.

Insurer Philamlife could have presented as witness its Medical The agents in short do what the company set them out to do.
Examiner Dr. Urbano Guinto. It was he who accomplished the
application, Part II, medical. Philamlife did not. The Insular Life case was decided some forty years ago when the
pressure of insurance salesmanship was not overwhelming as it is now;
Philamlife could have put to the witness stand its Agent Bienvenido S. when the population of this country was less than one-fourth of what it
Guinto, a relative to Dr. Guinto, Again Philamlife did not. (pp. 138139, is now; when the insurance companies competing with one another
Rollo) could be counted by the fingers. (pp. 140-142, Rollo)

xxx xxx xxx xxx xxx xxx

This Honorable Supreme Court has had occasion to denounce the In the face of all the above, it would be unjust if, having been subjected
pressure and practice indulged in by agents in selling insurance. At one to the whirlwind pressure of insurance salesmanship this Court itself has
time or another most of us have been subjected to that pressure, that long denounced, the assured who dies within the two-year period,
practice. This court took judicial cognizance of the whirlwind pressure should stand charged of fraudulent concealment and misrepresentation."
of insurance selling-especially of the agent's practice of 'supplying the (p. 142, Rollo)
information, preparing and answering the application, submitting the
application to their companies, concluding the transactions and The legislative answer to the arguments posed by the petitioners is the
otherwisesmoothing out all difficulties. "incontestability clause" added by the second paragraph of Section 48.

INSURANCE 26
The insurer has two years from the date of issuance of the insurance found to be diabetic and hypertensive; that by January, 1973, the
contract or of its last reinstatement within which to contest the policy, deceased was complaining of progressive weight loss and abdominal
whether or not, the insured still lives within such period. After two pain and was diagnosed to be suffering from hepatoma, (t.s.n. August
years, the defenses of concealment or misrepresentation, no matter how 23, 1976, pp. 8-10; Exhibit 2). Another physician, Dr. Wenceslao Vitug,
patent or well founded, no longer lie. Congress felt this was a sufficient testified that the deceased came to see him on December 14, 1973 for
answer to the various tactics employed by insurance companies to avoid consolation and claimed to have been diabetic for five years. (t.s.n.,
liability. The petitioners' interpretation would give rise to the Aug. 23,1976, p. 5; Exhibit 6) Because of the concealment made by the
incongruous situation where the beneficiaries of an insured who dies deceased of his consultations and treatments for hypertension, diabetes
right after taking out and paying for a life insurance policy, would be and liver disorders, respondent company was thus misled into accepting
allowed to collect on the policy even if the insured fraudulently the risk and approving his application as medically standard (Exhibit 5-
concealed material facts. C) and dispensing with further medical investigation and examination
(Exhibit 5-A). For as long as no adverse medical history is revealed in
The petitioners argue that no evidence was presented to show that the the application form, an applicant for insurance is presumed to be
medical terms were explained in a layman's language to the insured. healthy and physically fit and no further medical investigation or
They state that the insurer should have presented its two medical field examination is conducted by respondent company. (t.s.n., April 8,1976,
examiners as witnesses. Moreover, the petitioners allege that the policy pp. 6-8). (Rollo, pp. 96-98)
intends that the medical examination must be conducted before its
issuance otherwise the insurer "waives whatever imperfection by There is no strong showing that we should apply the "fine print" or
ratification." "contract of adhesion" rule in this case. (Sweet Lines, Inc. v. Teves, 83
SCRA 361 [1978]). The petitioners cite:
We agree with the Court of Appeals which ruled:
It is a matter of common knowledge that large amounts of money are
On the other hand, petitioners argue that no evidence was presented by collected from ignorant persons by companies and associations which
respondent company to show that the questions appearing in Part II of adopt high sounding titles and print the amount of benefits they agree to
the application for insurance were asked, explained to and understood pay in large black-faced type, following such undertakings by fine print
by the deceased so as to prove concealment on his part. The same is not conditions which destroy the substance of the promise. All provisions,
well taken. The deceased, by affixing his signature on the application conditions, or exceptions which in any way tend to work a forfeiture of
form, affirmed the correctness of all the entries and answers appearing the policy should be construed most strongly against those for whose
therein. It is but to be expected that he, a businessman, would not have benefit they are inserted, and most favorably toward those against
affixed his signature on the application form unless he clearly whom they are meant to operate. (Trinidad v. Orient Protective
understood its significance. For, the presumption is that a person intends Assurance Assn., 67 Phil. 184)
the ordinary consequence of his voluntary act and takes ordinary care of
his concerns. [Sec. 5(c) and (d), Rule 131, Rules of Court]. There is no showing that the questions in the application form for
insurance regarding the insured's medical history are in smaller print
The evidence for respondent company shows that on September than the rest of the printed form or that they are designed in such a way
19,1972, the deceased was examined by Dr. Victoriano Lim and was as to conceal from the applicant their importance. If a warning in bold
red letters or a boxed warning similar to that required for cigarette
INSURANCE 27
advertisements by the Surgeon General of the United States is > Tan Lee Siong was issued a policy by Philamlife on Nov. 6, 1973.
necessary, that is for Congress or the Insurance Commission to provide
as protection against high pressure insurance salesmanship. We are > On Aprl 26, 1975, Tan died of hepatoma. His beneficiaries then filed
limited in this petition to ascertaining whether or not the respondent a claim with Philamlife for the proceeds of the insurance.
Court of Appeals committed reversible error. It is the petitioners' burden
to show that the factual findings of the respondent court are not based > Philamlife wrote the beneficiaries in Sep. 1975 denying their claim
on substantial evidence or that its conclusions are contrary to applicable and rescinding the contract on the ground of misrepresentation. The
law and jurisprudence. They have failed to discharge that burden. beneficiaries contend that Philamlife can no longer rescind the contract
on the ground of misrepresentation as rescission must allegedly be done
WHEREFORE, the petition is hereby DENIED for lack of merit. The during the lifetime of the insured within two years and prior to the
questioned decision of the Court of Appeals is AFFIRMED. commencement of the action following the wording of Sec. 48, par. 2.

SO ORDERED. Issue:
Fernan, (C.J., Chairman), Bidin and Cortes, JJ., concur. Whether or not Philamlife can rescind the contract.

Feliciano, took no part. Held:

YES.

The phrase during the lifetime found in Sec. 48 simply means that
the policy is no longer in force after the insured has died. The key
phrase in the second paragraph is for a period of two years.

What is a simpler illustration of the ruling in Tan v. CA?

The period to consider in a life insurance poiicy is two years from


the date of issue or of the last reinstatement. So if for example the
Tan v. CA - Rescission of policy was issued/reinstated on Jan 1, 2000, the insurer can still exercise
his right to rescind up to Jan. 1, 2003 or two years from the date of
the contract of insurance issue/reinstatement, REGARDLESS of whether the insured died before
or after Jan. 1, 2003.
174 SCRA 403
Facts:
INSURANCE 28
Emilio Tan vs Court of After a policy of life insurance made payable on the
death of the insured shall have been in force during the
Appeals lifetime of the insured for a period of two years from the
date of its issue or of its last reinstatement, the insurer
cannot prove that the policy is void ab initio or is
rescindable by reason of the fraudulent concealment or
174 SCRA 403 Mercantile Law Insurance Law Representation misrepresentation of the insured or his agent.
Concealment Rescission of an Insurance Contract
The so-called incontestability clause precludes the insurer from
In September 1973, Tan Lee Siong applied for a life insurance under the raising the defenses of false representations or concealment of material
Philippine American Life Insurance Company (PHILAMLIFE). He facts insofar as health and previous diseases are concerned if the
stated in the application form that he has no health issues whatsoever insurance has been in force for at least two years during the insureds
and so in November 1973 he was issued a life insurance policy in the lifetime. The phrase during the lifetime found in Section 48 simply
amount of P80,000.00. He listed his sons as beneficiaries (Emilio Tan et means that the policy is no longer considered in force after the insured
al). In April 1975, Tan Lee Siong died due to hepatoma. His sons filed has died. The key phrase in the second paragraph of Section 48 is for
an insurance claim but PHILAMLIFE denied the same as it alleged that a period of two years.
Tan Lee Siong concealed the fact that he was hypertensive, diabetic, and
was suffering from hepatoma at the time of his application for the Note that the policy was in force for only one year and 5 months when
insurance. Tan Lee Siong died. This means that PHILAMLIFE can still contest and
rescind the policy issued by reason of the misrepresentation made by
The beneficiaries averred that PHILAMLIFE can no longer rescind the Tan Lee Siong.
insurance contract because the insured is already dead. They invoke
Section 48 of the Insurance Code which they interpreted to mean that an Further, because of Tan Lee Siongs statement that he does not have
insurer can only rescind an insurance contract during the lifetime of the any health issues, the insurance company was misled into believing that
insured; and that such rescission should be done within two years prior he was healthy and so it did not deem a medical checkup to be
to the filing of a suit involving the insurance. necessary and that ultimately led to the issuance of the life insurance
policy.
ISSUE: Whether or not the interpretation of the Tan brothers is correct.

HELD: No. The pertinent section in the Insurance Code provides:

Section 48. Whenever a right to rescind a contract of


insurance is given to the insurer by any provision of this
chapter, such right must be exercised previous to the
commencement of an action on the contract.

INSURANCE 29
Ratio:

The Insurance Code states in Section 48:

Whenever a right to rescind a contract of insurance is given to


the insurer by any provision of this chapter, such right must be
exercised previous to the commencement of an action on the
contract.

After a policy of life insurance made payable on the death of the


insured shall have been in force during the lifetime of the insured for
a period of two years from the date of its issue or of its last
reinstatement, the insurer cannot prove that the policy is void ab
Tan v CA G.R. No. 48049 June 29, 1989 initio or is rescindable by reason of the fraudulent concealment
or misrepresentation of the insured or his agent.
J. Gutierrez Jr.
The so-called "incontestability clause" in the second paragraph
prevents the insurer from raising the defenses of false
Facts: representations insofar as health and previous diseases are
concerned if the insurance has been in force for at least two years
Tan Lee Siong, father of the petitioners, applied for life insurance in during the insured's lifetime.
the amount of P 80,000.00 with Philamlife. It was approved. Tan Lee
Siong died of hepatoma. Petitioners then filed a claim for the The policy was in force for a period of only one year and five months.
proceeds. The company denied petitioners' claim and rescinded the Considering that the insured died before the two-year period had
policy by reason of the alleged misrepresentation and concealment lapsed, respondent company is not, therefore, barred from proving
of material facts. The premiums paid on the policy were refunded. that the policy is void ab initio by reason of the insured's fraudulent
The petitioners filed a complaint in the Insurance Commission. The concealment or misrepresentation.
latter dismissed the complaint.

The "incontestability clause" added by the second paragraph of


The Court of Appeals dismissed ' the petitioners' appeal from the Section 48 is in force for two years. After this, the defenses of
Insurance Commissioner's decision for lack of merit. Hence, this concealment or misrepresentation no longer lie.
petition.

The petitioners argue that no evidence was presented to show that


Issue: the medical terms were explained in a layman's language to the
insured. They also argue that no evidence was presented by
WON Philam didnt have the right to rescind the contract of respondent company to show that the questions appearing in Part II
insurance as rescission must allegedly be done during the lifetime of of the application for insurance were asked, explained to and
the insured within two years and prior to the commencement of understood by the deceased so as to prove concealment on his part.
action. This couldnt be accepted because the insured signed the form. He
affirmed the correctness of all the entries.
Held: No. Petition dismissed.
INSURANCE 30
The company records show that the deceased was examined by Dr. 1917, Luis Lim y Garcia of Zamboanga made application to the Sun
Victoriano Lim and was found to be diabetic and hypertensive. He Life Assurance Company of Canada for a policy of insurance on his life
was also found to have suffered from hepatoma. Because of the
concealment made by the deceased, the company was thus misled
in the sum of P5,000. In his application Lim designated his wife, Pilar
into accepting the risk and approving his application as medically C. de Lim, the plaintiff herein, as the beneficiary. The first premium of
fit. P433 was paid by Lim, and upon such payment the company issued
what was called a "provisional policy." Luis Lim y Garcia died on
August 23, 1917, after the issuance of the provisional policy but before
approval of the application by the home office of the insurance
company. The instant action is brought by the beneficiary, Pilar C. de
Lim, to recover from the Sun Life Assurance Company of Canada the
sum of P5,000, the amount named in the provisional policy.

The "provisional policy" upon which this action rests reads as follows:

Received (subject to the following stipulations and


agreements) the sum of four hundred and thirty-three
pesos, being the amount of the first year's premium for a
Life Assurance Policy on the life of Mr. Luis D. Lim y
G.R. No. L-15774 November 29, 1920 Garcia of Zamboanga for P5,000, for which an
application dated the 6th day of July, 1917, has been
PILAR C. DE LIM, plaintiff-appellant, made to the Sun Life Assurance Company of Canada.
vs.
SUN LIFE ASSURANCE COMPANY OF CANADA, defendant- The above-mentioned life is to be assured in accordance
appellee. with the terms and conditions contained or inserted by
the Company in the policy which may be granted by it in
Sanz and Luzuriaga for appellant. this particular case for four months only from the date of
Cohn and Fisher for appellee. the application, provided that the Company shall confirm
this agreement by issuing a policy on said application
MALCOLM, J.: when the same shall be submitted to the Head Office in
Montreal. Should the Company not issue such a policy,
This is an appeal by plaintiff from an order of the Court of First Instance then this agreement shall be null and void ab initio, and
of Zamboanga sustaining a demurrer to plaintiff's complaint upon the the Company shall be held not to have been on the risk at
ground that it fails to state a cause of action. all, but in such case the amount herein acknowledged
shall be returned.
As the demurrer had the effect of admitting the material facts set forth in
the complaint, the facts are those alleged by the plaintiff. On July 6,
INSURANCE 31
[SEAL.] (Sgd.) T. B. MACAULAY, President. Appellant relies on Joyce on Insurance. Beginning at page 253, of
(Sgd.) A. F. Peters, Agent. Volume I, Joyce states the general rule concerning the agent's receipt
pending approval or issuance of policy. The first rule which Joyce lays
Our duty in this case is to ascertain the correct meaning of the document down is this: If the act of acceptance of the risk by the agent and the
above quoted. A perusal of the same many times by the writer and by giving by him of a receipt, is within the scope of the agent's authority,
other members of the court leaves a decided impression of vagueness in and nothing remains but to issue a policy, then the receipt will bind the
the mind. Apparently it is to be a provisional policy "for four months company. This rule does not apply, for while here nothing remained but
only from the date of this application." We use the term "apparently" to issue the policy, this was made an express condition to the contract.
advisedly, because immediately following the words fixing the four The second rule laid down by Joyce is this: Where an agreement is
months period comes the word "provided" which has the meaning of made between the applicant and the agent whether by signing an
"if." Otherwise stated, the policy for four months is expressly made application containing such condition, or otherwise, that no liability
subjected to the affirmative condition that "the company shall confirm shall attach until the principal approves the risk and a receipt is given
this agreement by issuing a policy on said application when the same buy the agent, such acceptance is merely conditional, and it
shall be submitted to the head office in Montreal." To reenforce the subordinated to the act of the company in approving or rejecting; so in
same there follows the negative condition life insurance a "binding slip" or "binding receipt" does not insure of
itself. This is the rule which we believe applies to the instant case. The
Should the company not issue such a policy, then this agreement shall third rule announced by Joyce is this: Where the acceptance by the
be null and void ab initio, and the company shall be held not to have agent is within the scope of his authority a receipt containing a contract
been on the risk." Certainly, language could hardly be used which would for insurance for a specific time which is not absolute but conditional,
more clearly stipulate that the agreement should not go into effect until upon acceptance or rejection by the principal, covers the specified
the home office of the company should confirm it by issuing a policy. period unless the risk is declined within that period. The case cited by
As we read and understand the so-called provisional policy it amounts Joyce to substantiate the last principle is that a Goodfellow vs. Times &
to nothing but an acknowledgment on behalf of the company, that it has Beacon Assurance Com. (17 U. C. Q. B., 411), not available.
received from the person named therein the sum of money agreed upon
as the first year's premium upon a policy to be issued upon the The two cases most nearly in point come from the federal courts and the
application, if the application is accepted by the company. Supreme Court of Arkansas.

It is of course a primary rule that a contract of insurance, like other In the case of Steinle vs. New York Life Insurance Co. ([1897], 81 Fed.,
contracts, must be assented to by both parties either in person or by their 489} the facts were that the amount of the first premium had been paid
agents. So long as an application for insurance has not been either to an insurance agent and a receipt given therefor. The receipt, however,
accepted or rejected, it is merely an offer or proposal to make a contract. expressly declared that if the application was accepted by the company,
The contract, to be binding from the date of the application, must have the insurance shall take effect from the date of the application but that if
been a completed contract, one that leaves nothing to be done, nothing the application was not accepted, the money shall be returned. The trite
to be completed, nothing to be passed upon, or determined, before it decision of the circuit court of appeal was, "On the conceded facts of
shall take effect. There can be no contract of insurance unless the minds this case, there was no contract to life insurance perfected and the
of the parties have met in agreement. Our view is, that a contract of judgment of the circuit court must be affirmed."
insurance was not here consummated by the parties.lawph!l.net
INSURANCE 32
In the case of Cooksey vs. Mutual Life Insurance Co. ([1904], 73 Ark., for the return of the first premium to the estate of the deceased. It is not
117) the person applying for the life insurance paid and amount equal to to be doubted but that the Sun Life Assurance Company of Canada will
the first premium, but the application and the receipt for the money immediately, on the promulgation of this decision, pay to the estate of
paid, stipulated that the insurance was to become effective only when the late Luis Lim y Garcia the of P433.
the application was approved and the policy issued. The court held that
the transaction did not amount to an agreement for preliminary or The order appealed from, in the nature of a final judgment is affirmed,
temporary insurance. It was said: without special finding as to costs in this instance. So ordered.

It is not an unfamiliar custom among life insurance companies in the Mapa, C.J., Johnson, Araullo, Avancea and Villamor, JJ., concur.
operation of the business, upon receipt of an application for insurance,
to enter into a contract with the applicant in the shape of a so-called
"binding receipt" for temporary insurance pending the consideration of Lim v. Sun Life
the application, to last until the policy be issued or the application
rejected, and such contracts are upheld and enforced when the applicant
dies before the issuance of a policy or final rejection of the application.
41 PHIL 263
It is held, too, that such contracts may rest in parol. Counsel for
appellant insists that such a preliminary contract for temporary Facts:
insurance was entered into in this instance, but we do not think so. On
the contrary, the clause in the application and the receipt given by the > On July 6, 1917, Luis Lim Y Garcia of Zamboanga applied for a
solicitor, which are to be read together, stipulate expressly that the policy of life insurance with Sunlife in the amount of 5T.
insurance shall become effective only when the "application shall be
approved and the policy duly signed by the secretary at the head office > He designated his wife Pilar Lim as the beneficiary. The first
of the company and issued." It constituted no agreement at all for premium of P433 was paid by Lim and company issued a provisional
preliminary or temporary insurance; Mohrstadt vs. Mutual Life Ins. Co., policy
115 Fed., 81, 52 C. C. A., 675; Steinle vs. New York Life Ins. Co., 81
Fed., 489, 26 C. C. A., 491." (See further Weinfeld vs. Mutual Reserve > Such policy contained the following provisions xx the
Fund Life Ass'n. [1892], 53 Fed, 208' Mohrstadt vs. Mutual Life abovementioned life is to be assured in accordance with the terms and
Insurance Co. [1902], 115 Fed., 81; Insurance co. vs. Young's conditions contained or inserted by the Company in the policy which
Administrator [1875], 90 U. S., 85; Chamberlain vs. Prudential may be granted by it in this particular case for 4 months only from the
Insurance Company of America [1901], 109 Wis., 4; Shawnee Mut. Fire date of the application, PROVIDED that the company shall confirm this
Ins. Co. vs. McClure [1913], 39 Okla., 509; Dorman vs. Connecticut agreement by issuing a policy on said application xxx. Should the
Fire Ins. Co. [1914], 51 contra, Starr vs. Mutual Life Ins. Co. [1905], 41 company NOT issue such a policy, then this agreement shall be null and
Wash., 228.) void ab initio and the Company shall be held not to have been on the
risk at all, but in such case, the amount herein shall be returned.
We are of the opinion that the trial court committed no error in
sustaining the demurrer and dismissing the case. It is to be noted,
however, that counsel for appellee admits the liability of the company
INSURANCE 33
> Lim died on Aug. 23, 1917 after the issuance of the provisional
policy but before the approval of the application by the home office of
the insurance company.

> The instant action is brought by the beneficiary to recover from Sun
Life the sum of 5T.

Issue:

Whether or not the beneficiary can collect the 5T.

Held: G.R. No. L-31845 April 30, 1979

NO. GREAT PACIFIC LIFE ASSURANCE COMPANY, petitioner,


vs.
The contract of insurance was not consummated by the parties. The HONORABLE COURT OF APPEALS, respondents.
above quoted agreement clearly stated that the agreement should NOT
go into effect until the home office of the Company shall confirm it by G.R. No. L-31878 April 30, 1979
issuing a policy. It was nothing but an acknowledgment by the
Company that it has received a sum of money agreed upon as the first LAPULAPU D. MONDRAGON, petitioner,
years premium upon a policy to be issued upon the application if it is vs.
accepted by the Company. HON. COURT OF APPEALS and NGO HING, respondents.

When an agreement is made between the applicant and the agent Siguion Reyna, Montecillo & Ongsiako and Sycip, Salazar, Luna &
whether by signing an application containing such condition or Manalo for petitioner Company.
otherwise, that no liability shall attach until the principal approves the
risk and a receipt is given by the agent, such acceptance is merely Voltaire Garcia for petitioner Mondragon.
conditional and is subordinated to the companys act in approving or
rejecting; so in life insurance a binding slip or receipt does not
Pelaez, Pelaez & Pelaez for respondent Ngo Hing.
insure itself.
DE CASTRO, J.:

INSURANCE 34
The two above-entitled cases were ordered consolidated by the The non-acceptance of the insurance plan by Pacific Life was allegedly
Resolution of this Court dated April 29, 1970, (Rollo, No. L-31878, p. not communicated by petitioner Mondragon to private respondent Ngo
58), because the petitioners in both cases seek similar relief, through Hing. Instead, on May 6, 1957, Mondragon wrote back Pacific Life
these petitions for certiorari by way of appeal, from the amended again strongly recommending the approval of the 20-year endowment
decision of respondent Court of Appeals which affirmed in toto the insurance plan to children, pointing out that since 1954 the customers,
decision of the Court of First Instance of Cebu, ordering "the defendants especially the Chinese, were asking for such coverage (Exhibit 4-M).
(herein petitioners Great Pacific Ligfe Assurance Company and
Mondragon) jointly and severally to pay plaintiff (herein private It was when things were in such state that on May 28, 1957 Helen Go
respondent Ngo Hing) the amount of P50,000.00 with interest at 6% died of influenza with complication of bronchopneumonia. Thereupon,
from the date of the filing of the complaint, and the sum of P1,077.75, private respondent sought the payment of the proceeds of the insurance,
without interest. but having failed in his effort, he filed the action for the recovery of the
same before the Court of First Instance of Cebu, which rendered the
It appears that on March 14, 1957, private respondent Ngo Hing filed an adverse decision as earlier refered to against both petitioners.
application with the Great Pacific Life Assurance Company (hereinafter
referred to as Pacific Life) for a twenty-year endownment policy in the The decisive issues in these cases are: (1) whether the binding deposit
amount of P50,000.00 on the life of his one-year old daughter Helen receipt (Exhibit E) constituted a temporary contract of the life insurance
Go. Said respondent supplied the essential data which petitioner in question; and (2) whether private respondent Ngo Hing concealed the
Lapulapu D. Mondragon, Branch Manager of the Pacific Life in Cebu state of health and physical condition of Helen Go, which rendered void
City wrote on the corresponding form in his own handwriting (Exhibit the aforesaid Exhibit E.
I-M). Mondragon finally type-wrote the data on the application form
which was signed by private respondent Ngo Hing. The latter paid the 1. At the back of Exhibit E are condition precedents required before a
annual premuim the sum of P1,077.75 going over to the Company, but deposit is considered a BINDING RECEIPT. These conditions state
he reatined the amount of P1,317.00 as his commission for being a duly that:
authorized agebt of Pacific Life. Upon the payment of the insurance
premuim, the binding deposit receipt (Exhibit E) was issued to private A. If the Company or its agent, shan have received the premium
respondent Ngo Hing. Likewise, petitioner Mondragon handwrote at the deposit ... and the insurance application, ON or PRIOR to the date of
bottom of the back page of the application form his strong medical examination ... said insurance shan be in force and in
recommendation for the approval of the insurance application. Then on effect from the date of such medical examination, for such period as is
April 30, 1957, Mondragon received a letter from Pacific Life covered by the deposit ...,PROVIDED the company shall be satisfied
disapproving the insurance application (Exhibit 3-M). The letter stated that on said date the applicant was insurable on standard rates under
that the said life insurance application for 20-year endowment plan is its rule for the amount of insurance and the kind of policy requested in
not available for minors below seven years old, but Pacific Life can the application.
consider the same under the Juvenile Triple Action Plan, and advised
that if the offer is acceptable, the Juvenile Non-Medical Declaration be
D. If the Company does not accept the application on standard rate for
sent to the company.
the amount of insurance and/or the kind of policy requested in the
application but issue, or offers to issue a policy for a different plan
INSURANCE 35
and/or amount ..., the insurance shall not be in force and in effect until where an agreement is made between the applicant and the agent, no
the applicant shall have accepted the policy as issued or offered by the liability shall attach until the principal approves the risk and a receipt is
Company and shall have paid the full premium thereof. If the applicant given by the agent. The acceptance is merely conditional and is
does not accept the policy, the deposit shall be refunded. subordinated to the act of the company in approving or rejecting the
application. Thus, in life insurance, a "binding slip" or "binding receipt"
E. If the applicant shall not have been insurable under Condition A does not insure by itself (De Lim vs. Sun Life Assurance Company of
above, and the Company declines to approve the application the Canada, 41 Phil. 264).
insurance applied for shall not have been in force at any time and the
sum paid be returned to the applicant upon the surrender of this It bears repeating that through the intra-company communication of
receipt. (Emphasis Ours). April 30, 1957 (Exhibit 3-M), Pacific Life disapproved the insurance
application in question on the ground that it is not offering the twenty-
The aforequoted provisions printed on Exhibit E show that the binding year endowment insurance policy to children less than seven years of
deposit receipt is intended to be merely a provisional or temporary age. What it offered instead is another plan known as the Juvenile Triple
insurance contract and only upon compliance of the following Action, which private respondent failed to accept. In the absence of a
conditions: (1) that the company shall be satisfied that the applicant was meeting of the minds between petitioner Pacific Life and private
insurable on standard rates; (2) that if the company does not accept the respondent Ngo Hing over the 20-year endowment life insurance in the
application and offers to issue a policy for a different plan, the insurance amount of P50,000.00 in favor of the latter's one-year old daughter, and
contract shall not be binding until the applicant accepts the policy with the non-compliance of the abovequoted conditions stated in the
offered; otherwise, the deposit shall be reftmded; and (3) that if the disputed binding deposit receipt, there could have been no insurance
applicant is not ble according to the standard rates, and the company contract duly perfected between thenl Accordingly, the deposit paid by
disapproves the application, the insurance applied for shall not be in private respondent shall have to be refunded by Pacific Life.
force at any time, and the premium paid shall be returned to the
applicant. As held in De Lim vs. Sun Life Assurance Company of Canada, supra,
"a contract of insurance, like other contracts, must be assented to by
Clearly implied from the aforesaid conditions is that the binding deposit both parties either in person or by their agents ... The contract, to be
receipt in question is merely an acknowledgment, on behalf of the binding from the date of the application, must have been a completed
company, that the latter's branch office had received from the applicant contract, one that leaves nothing to be dione, nothing to be completed,
the insurance premium and had accepted the application subject for nothing to be passed upon, or determined, before it shall take effect.
processing by the insurance company; and that the latter will either There can be no contract of insurance unless the minds of the parties
approve or reject the same on the basis of whether or not the applicant is have met in agreement."
"insurable on standard rates." Since petitioner Pacific Life disapproved
the insurance application of respondent Ngo Hing, the binding deposit We are not impressed with private respondent's contention that failure of
receipt in question had never become in force at any time. petitioner Mondragon to communicate to him the rejection of the
insurance application would not have any adverse effect on the
Upon this premise, the binding deposit receipt (Exhibit E) is, manifestly, allegedly perfected temporary contract (Respondent's Brief, pp. 13-14).
merely conditional and does not insure outright. As held by this Court, In this first place, there was no contract perfected between the parties
who had no meeting of their minds. Private respondet, being an
INSURANCE 36
authorized insurance agent of Pacific Life at Cebu branch office, is Hing himself who insisted on the application as originally filed, thereby
indubitably aware that said company does not offer the life insurance implictly declining the offer to consider the application under the
applied for. When he filed the insurance application in dispute, private Juvenile Triple Action Plan. Besides, the associate of Mondragon that he
respondent was, therefore, only taking the chance that Pacific Life will was, Ngo Hing should only be presumed to know what kind of policies
approve the recommendation of Mondragon for the acceptance and are available in the company for minors below 7 years old. What he and
approval of the application in question along with his proposal that the Mondragon were apparently trying to do in the premises was merely to
insurance company starts to offer the 20-year endowment insurance plan prod the company into going into the business of issuing endowment
for children less than seven years. Nonetheless, the record discloses that policies for minors just as other insurance companies allegedly do. Until
Pacific Life had rejected the proposal and recommendation. Secondly, such a definite policy is however, adopted by the company, it can hardly
having an insurable interest on the life of his one-year old daughter, be said that it could have been bound at all under the binding slip for a
aside from being an insurance agent and an offense associate of plan of insurance that it could not have, by then issued at all. (Amended
petitioner Mondragon, private respondent Ngo Hing must have known Decision, Rollo, pp- 52-53).
and followed the progress on the processing of such application and
could not pretend ignorance of the Company's rejection of the 20-year 2. Relative to the second issue of alleged concealment. this Court is of
endowment life insurance application. the firm belief that private respondent had deliberately concealed the
state of health and piysical condition of his daughter Helen Go. Wher
At this juncture, We find it fit to quote with approval, the very apt private regpondeit supplied the required essential data for the insurance
observation of then Appellate Associate Justice Ruperto G. Martin who application form, he was fully aware that his one-year old daughter is
later came up to this Court, from his dissenting opinion to the amended typically a mongoloid child. Such a congenital physical defect could
decision of the respondent court which completely reversed the original never be ensconced nor disguished. Nonetheless, private respondent, in
decision, the following: apparent bad faith, withheld the fact materal to the risk to be assumed
by the insurance compary. As an insurance agent of Pacific Life, he
Of course, there is the insinuation that neither the memorandum of ought to know, as he surely must have known. his duty and
rejection (Exhibit 3-M) nor the reply thereto of appellant Mondragon responsibility to such a material fact. Had he diamond said significant
reiterating the desire for applicant's father to have the application fact in the insurance application fom Pacific Life would have verified
considered as one for a 20-year endowment plan was ever duly the same and would have had no choice but to disapprove the
communicated to Ngo; Hing, father of the minor applicant. I am not application outright.
quite conninced that this was so. Ngo Hing, as father of the applicant
herself, was precisely the "underwriter who wrote this case" (Exhibit H- The contract of insurance is one of perfect good faith uberrima fides
1). The unchallenged statement of appellant Mondragon in his letter of meaning good faith, absolute and perfect candor or openness and
May 6, 1957) (Exhibit 4-M), specifically admits that said Ngo Hing was honesty; the absence of any concealment or demotion, however slight
"our associate" and that it was the latter who "insisted that the plan be [Black's Law Dictionary, 2nd Edition], not for the alone but equally so
placed on the 20-year endowment plan." Under these circumstances, it for the insurer (Field man's Insurance Co., Inc. vs. Vda de Songco, 25
is inconceivable that the progress in the processing of the application SCRA 70). Concealment is a neglect to communicate that which a partY
was not brought home to his knowledge. He must have been duly knows aDd Ought to communicate (Section 25, Act No. 2427). Whether
apprised of the rejection of the application for a 20-year endowment intentional or unintentional the concealment entitles the insurer to
plan otherwise Mondragon would not have asserted that it was Ngo rescind the contract of insurance (Section 26, Id.: Yu Pang Cheng vs.
INSURANCE 37
Court of Appeals, et al, 105 Phil 930; Satumino vs. Philippine American In March 1957, Ngo Hing filed an application for a 20-year endowment
Life Insurance Company, 7 SCRA 316). Private respondent appears policy for the life of his one-year old daughter with the Great Pacific
guilty thereof. Life Assurance Company (Grepalife). Lapulapu Mondragon was the
insurance agent who assisted Ngo Hing. The insurance policy was for
We are thus constrained to hold that no insurance contract was perfected P50,000.00. The proper form was filled out and Ngo Hing paid the
between the parties with the noncompliance of the conditions provided insurance premium. He received a binding deposit receipt in return. Said
in the binding receipt, and concealment, as legally defined, having been receipt however was subject to certain conditions, among which is the
comraitted by herein private respondent. acceptance of Grepalife.

WHEREFORE, the decision appealed from is hereby set aside, and in Grepalife eventually denied the insurance application because the
lieu thereof, one is hereby entered absolving petitioners Lapulapu D. endowment plan by Grepalife is not offered for minors below seven
Mondragon and Great Pacific Life Assurance Company from their civil years old. Grepalife, instead made a counter-offer which Ngo Hing
liabilities as found by respondent Court and ordering the aforesaid failed to accept because Mondragon, instead of communicating the said
insurance company to reimburse the amount of P1,077.75, without denial to Ngo Hing, wrote a letter to Grepalife trying to convince
interest, to private respondent, Ngo Hing. Costs against private Grepalife to allow one-year olds to be covered by endowment plans.
respondent.
In May 1957, Ngo Hings one-year old daughter died. Ngo Hing tried
SO ORDERED. to collect the insurance claim but Grepalife refused as it claimed that the
insurance contract was never perfected sans their acceptance.
Teehankee (Chairman), Makasiar, Guerrero and Melencio-Herrera, JJ.,
concur. ISSUE: Whether or not Grepalife should pay the insurance claim.

Fernandez, J., took no part. HELD: No. As properly ruled by the lower court as well as the Court of
Appeals, the insurance contract was never completed because Grepalife
never accepted the insurance offer. The binding deposit receipt issued to
Ngo Hing is only acknowledgement of his application and receipt of his
payment for the insurance premium.
Great Pacific Life
Assurance Company vs The Supreme Court also noted that Ngo Hing failed to disclose the fact
that his one-year old daughter was a mongoloid. Such congenital defect
Court of Appeals (1979) was withheld by Ngo Hing with bad faith and such risk to be assumed
by the insurance company.
89 SCRA 543 Mercantile Law Insurance Law Concealment
Insurance Contract as an Uberrima Fides Contract The contract of insurance is one of perfect good faith uberrima fides
meaning good faith, absolute and perfect candor or openness and
honesty; the absence of any concealment or demotion, however slight
INSURANCE 38
not for the insured alone but equally so for the insurer. Concealment is a the sum of P3,000.00 as attorney's fees and the costs 1 thereby
neglect to communicate that which a party knows and ought to dismissing petitioner s complaint with costs. 2
communicate. Whether intentional or unintentional the concealment
entitles the insurer to rescind the contract of insurance. The findings of the of fact of the Court of Appeals, which are generally
binding upon this Court, Except as shall be indicated in the discussion
of the opinion of this Court the substantial correctness of still particular
finding having been disputed, thereby raising a question of law
reviewable by this Court 3 are as follows:

March 19, l963, the plaintiff secured temporary insurance from the
defendant for its exportation of 1,250,000 board feet of Philippine
Lauan and Apitong logs to be shipped from the Diapitan. Bay, Quezon
Province to Okinawa and Tokyo, Japan. The defendant issued on said
date Cover Note No. 1010, insuring the said cargo of the plaintiff
"Subject to the Terms and Conditions of the WORKMEN'S
INSURANCE COMPANY, INC. printed Marine Policy form as filed
with and approved by the Office of the Insurance Commissioner
(Exhibit A).

The regular marine cargo policies were issued by the defendant in favor
of the plaintiff on April 2, 1963. The two marine policies bore the
numbers 53 HO 1032 and 53 HO 1033 (Exhibits B and C, respectively).
Policy No. 53 H0 1033 (Exhibit B) was for 542 pieces of logs
G.R. No. L-38613 February 25, 1982
equivalent to 499,950 board feet. Policy No. 53 H0 1033 was for 853
pieces of logs equivalent to 695,548 board feet (Exhibit C). The total
PACIFIC TIMBER EXPORT CORPORATION, petitioner, cargo insured under the two marine policies accordingly consisted of
vs. 1,395 logs, or the equivalent of 1,195.498 bd. ft.
THE HONORABLE COURT OF APPEALS and WORKMEN'S
INSURANCE COMPANY, INC., respondents.
After the issuance of Cover Note No. 1010 (Exhibit A), but before the
issuance of the two marine policies Nos. 53 HO 1032 and 53 HO 1033,
DE CASTRO, ** J.: some of the logs intended to be exported were lost during loading
operations in the Diapitan Bay. The logs were to be loaded on the 'SS
This petition seeks the review of the decision of the Court of Appeals Woodlock' which docked about 500 meters from the shoreline of the
reversing the decision of the Court of First Instance of Manila in favor Diapitan Bay. The logs were taken from the log pond of the plaintiff and
of petitioner and against private respondent which ordered the latter to from which they were towed in rafts to the vessel. At about 10:00
pay the sum of Pll,042.04 with interest at the rate of 12% interest from o'clock a. m. on March 29, 1963, while the logs were alongside the
receipt of notice of loss on April 15, 1963 up to the complete payment,
INSURANCE 39
vessel, bad weather developed resulting in 75 pieces of logs which were loss under Policies Nos. 53 HO 1032 and 53 HO 1033, in the total
rafted together co break loose from each other. 45 pieces of logs were amount of P19,286.79 (Exhibit G).
salvaged, but 30 pieces were verified to have been lost or washed away
as a result of the accident. On July 17, 1963, the defendant requested the First Philippine
Adjustment Corporation to inspect the loss and assess the damage. The
In a letter dated April 4, 1963, the plaintiff informed the defendant about adjustment company submitted its 'Report on August 23, 1963 (Exhibit
the loss of 'appropriately 32 pieces of log's during loading of the 'SS H). In said report, the adjuster found that 'the loss of 30 pieces of logs is
Woodlock'. The said letter (Exhibit F) reads as follows: not covered by Policies Nos. 53 HO 1032 and 1033 inasmuch as said
policies covered the actual number of logs loaded on board the 'SS
April 4, 1963 Woodlock' However, the loss of 30 pieces of logs is within the
1,250,000 bd. ft. covered by Cover Note 1010 insured for $70,000.00.
Workmen's Insurance Company, Inc. Manila, Philippines
On September 14, 1963, the adjustment company submitted a
Gentlemen: computation of the defendant's probable liability on the loss sustained
by the shipment, in the total amount of Pl1,042.04 (Exhibit 4).
This has reference to Insurance Cover Note No. 1010 for shipment of
1,250,000 bd. ft. Philippine Lauan and Apitong Logs. We would like to On January 13, 1964, the defendant wrote the plaintiff denying the
inform you that we have received advance preliminary report from our latter's claim, on the ground they defendant's investigation revealed that
Office in Diapitan, Quezon that we have lost approximately 32 pieces of the entire shipment of logs covered by the two marines policies No. 53
logs during loading of the SS Woodlock. 110 1032 and 713 HO 1033 were received in good order at their point of
destination. It was further stated that the said loss may be considered as
We will send you an accurate report all the details including values as covered under Cover Note No. 1010 because the said Note had become
soon as same will be reported to us. 'null and void by virtue of the issuance of Marine Policy Nos. 53 HO
1032 and 1033'(Exhibit J-1). The denial of the claim by the defendant
was brought by the plaintiff to the attention of the Insurance
Thank you for your attention, we wish to remain.
Commissioner by means of a letter dated March 21, 1964 (Exhibit K).
In a reply letter dated March 30, 1964, Insurance Commissioner
Very respectfully yours, Francisco Y. Mandanas observed that 'it is only fair and equitable to
indemnify the insured under Cover Note No. 1010', and advised early
PACIFIC TIMBER EXPORT CORPORATION settlement of the said marine loss and salvage claim (Exhibit L).

(Sgd.) EMMANUEL S. ATILANO Asst. General Manager. On June 26, 1964, the defendant informed the Insurance Commissioner
that, on advice of their attorneys, the claim of the plaintiff is being
Although dated April 4, 1963, the letter was received in the office of the denied on the ground that the cover note is null and void for lack of
defendant only on April 15, 1963, as shown by the stamp impression valuable consideration (Exhibit M). 4
appearing on the left bottom corner of said letter. The plaintiff
subsequently submitted a 'Claim Statement demanding payment of the
INSURANCE 40
Petitioner assigned as errors of the Court of Appeals, the following: by the nature of the Cover Note, it did not contain, as all Cover Notes
do not contain particulars of the shipment that would serve as basis for
I the computation of the premiums. As a logical consequence, no separate
premiums are intended or required to be paid on a Cover Note. This is a
THE COURT OF APPEALS ERRED IN HOLDING THAT THE fact admitted by an official of respondent company, Juan Jose Camacho,
COVER NOTE WAS NULL AND VOID FOR LACK OF VALUABLE in charge of issuing cover notes of the respondent company (p. 33, tsn,
CONSIDERATION BECAUSE THE COURT DISREGARDED THE September 24, 1965).
PROVEN FACTS THAT PREMIUMS FOR THE COMPREHENSIVE
INSURANCE COVERAGE THAT INCLUDED THE COVER NOTE At any rate, it is not disputed that petitioner paid in full all the premiums
WAS PAID BY PETITIONER AND THAT INCLUDED THE COVER as called for by the statement issued by private respondent after the
NOTE WAS PAID BY PETITIONER AND THAT NO SEPARATE issuance of the two regular marine insurance policies, thereby leaving
PREMIUMS ARE COLLECTED BY PRIVATE RESPONDENT ON no account unpaid by petitioner due on the insurance coverage, which
ALL ITS COVER NOTES. must be deemed to include the Cover Note. If the Note is to be treated
as a separate policy instead of integrating it to the regular policies
II subsequently issued, the purpose and function of the Cover Note would
be set at naught or rendered meaningless, for it is in a real sense a
THE COURT OF APPEALS ERRED IN HOLDING THAT PRIVATE contract, not a mere application for insurance which is a mere offer. 6
RESPONDENT WAS RELEASED FROM LIABILITY UNDER THE
COVER NOTE DUE TO UNREASONABLE DELAY IN GIVING It may be true that the marine insurance policies issued were for logs no
NOTICE OF LOSS BECAUSE THE COURT DISREGARDED THE longer including those which had been lost during loading operations.
PROVEN FACT THAT PRIVATE RESPONDENT DID NOT This had to be so because the risk insured against is not for loss during
PROMPTLY AND SPECIFICALLY OBJECT TO THE CLAIM ON operations anymore, but for loss during transit, the logs having already
THE GROUND OF DELAY IN GIVING NOTICE OF LOSS AND, been safely placed aboard. This would make no difference, however,
CONSEQUENTLY, OBJECTIONS ON THAT GROUND ARE insofar as the liability on the cover note is concerned, for the number or
WAIVED UNDER SECTION 84 OF THE INSURANCE ACT. 5 volume of logs lost can be determined independently as in fact it had
been so ascertained at the instance of private respondent itself when it
1. Petitioner contends that the Cover Note was issued with a sent its own adjuster to investigate and assess the loss, after the issuance
consideration when, by express stipulation, the cover note is made of the marine insurance policies.
subject to the terms and conditions of the marine policies, and the
payment of premiums is one of the terms of the policies. From this The adjuster went as far as submitting his report to respondent, as well
undisputed fact, We uphold petitioner's submission that the Cover Note as its computation of respondent's liability on the insurance coverage.
was not without consideration for which the respondent court held the This coverage could not have been no other than what was stipulated in
Cover Note as null and void, and denied recovery therefrom. The fact the Cover Note, for no loss or damage had to be assessed on the
that no separate premium was paid on the Cover Note before the loss coverage arising from the marine insurance policies. For obvious
insured against occurred, does not militate against the validity of reasons, it was not necessary to ask petitioner to pay premium on the
petitioner's contention, for no such premium could have been paid, since Cover Note, for the loss insured against having already occurred, the
more practical procedure is simply to deduct the premium from the
INSURANCE 41
amount due the petitioner on the Cover Note. The non-payment of the assumption that there was delay, this Court is satisfied and
premium on the Cover Note is, therefore, no cause for the petitioner to convinced that as expressly provided by law, waiver can successfully be
lose what is due it as if there had been payment of premium, for non- raised against private respondent. Thus Section 84 of the Insurance Act
payment by it was not chargeable against its fault. Had all the logs been provides:
lost during the loading operations, but after the issuance of the Cover
Note, liability on the note would have already arisen even before Section 84.Delay in the presentation to an insurer of notice or proof
payment of premium. This is how the cover note as a "binder" should of loss is waived if caused by any act of his or if he omits to take
legally operate otherwise, it would serve no practical purpose in the objection promptly and specifically upon that ground.
realm of commerce, and is supported by the doctrine that where a policy
is delivered without requiring payment of the premium, the presumption From what has been said, We find duly substantiated petitioner's
is that a credit was intended and policy is valid. 7 assignments of error.

2. The defense of delay as raised by private respondent in resisting the ACCORDINGLY, the appealed decision is set aside and the decision of
claim cannot be sustained. The law requires this ground of delay to be the Court of First Instance is reinstated in toto with the affirmance of
promptly and specifically asserted when a claim on the insurance this Court. No special pronouncement as to costs.
agreement is made. The undisputed facts show that instead of invoking
the ground of delay in objecting to petitioner's claim of recovery on the SO ORDERED.
cover note, it took steps clearly indicative that this particular ground for
objection to the claim was never in its mind. The nature of this specific
Teehankee (Chairman), Makasiar, Fernandez Guerrero, Melencio-
ground for resisting a claim places the insurer on duty to inquire when
Herrera and Plana, JJ., concur.
the loss took place, so that it could determine whether delay would be a
valid ground upon which to object to a claim against it.

As already stated earlier, private respondent's reaction upon receipt of


the notice of loss, which was on April 15, 1963, was to set in motion
from July 1963 what would be necessary to determine the cause and
extent of the loss, with a view to the payment thereof on the insurance
agreement. Thus it sent its adjuster to investigate and assess the loss in
July, 1963. The adjuster submitted his report on August 23, 1963 and its
computation of respondent's liability on September 14, 1963. From
April 1963 to July, 1963, enough time was available for private
respondent to determine if petitioner was guilty of delay in
communicating the loss to respondent company. In the proceedings that
took place later in the Office of the Insurance Commissioner, private
respondent should then have raised this ground of delay to avoid
liability. It did not do so. It must be because it did not find any delay, as
this Court fails to find a real and substantial sign thereof. But even on
INSURANCE 42
Pacific Timber v CA G.R. No. L-38613 February Woodmens requested an adjustment company to assess the
damage. It submitted its report, where it found that the loss of 30
25, 1982 pieces of logs is not covered by Policies Nos. 53 HO 1032 and 1033
J. De Castro
but within the 1,250,000 bd. ft. covered by Cover Note 1010 insured
for $70,000.00.
Facts:
The adjustment company submitted a computation of the
The plaintiff secured temporary insurance from the defendant for its defendant's probable liability on the loss sustained by the shipment,
exportation of 1,250,000 board feet of Philippine Lauan and Apitong in the total amount of P11,042.04.
logs to be shipped from Quezon Province to Okinawa and
Tokyo, Japan.
Woodmens wrote the plaintiff denying the latter's claim on the
ground they defendant's investigation revealed that the entire
Workmens Insurance issued a cover note insuring the cargo of the shipment of logs covered by the two marine policies were received in
plaintiff subject to its terms and conditions. good order at their point of destination. It was further stated that the
said loss may be considered as covered under Cover Note No. 1010
The two marine policies bore the numbers 53 HO 1032 and 53 HO because the said Note had become null and void by virtue of the
1033. Policy No. 53 H0 1033 was for 542 pieces of logs equivalent to issuance of Marine Policy Nos. 53 HO 1032 and 1033.
499,950 board feet. Policy No. 53 H0 1033 was for 853 pieces of logs
equivalent to 695,548 board feet. The total cargo insured under the The denial of the claim by the defendant was brought by the plaintiff
two marine policies consisted of 1,395 logs, or the equivalent of to the attention of the Insurance Commissioner. The Insurance
1,195.498 bd. ft. Commissioner ruled in favor of indemnifying Pacific Timber. The
company added that the cover note is null and void for lack of
After the issuance of the cover note, but before the issuance of the valuable consideration. The trial court ruled in petitioners favor
two marine policies Nos. 53 HO 1032 and 53 HO 1033, some of the while the CA dismissed the case. Hence this appeal.
logs intended to be exported were lost during loading operations in
the Diapitan Bay. Issues:

While the logs were alongside the vessel, bad weather developed WON the cover note was null and void for lack of valuable
resulting in 75 pieces of logs which were rafted together co break consideration
loose from each other. 45 pieces of logs were salvaged, but 30
pieces were verified to have been lost or washed away as a result of
WON the Insurance company was absolved from responsibility due to
the accident.
unreasonable delay in giving notice of loss.

Pacific Timber informed Workmens about the loss of 32 pieces of


Held: No. No. Judgment reversed.
logs during loading of SS woodlock.

Ratio:
Although dated April 4, 1963, the letter was received in the office of
the defendant only on April 15, 1963. The plaintiff claimed for
insurance to the value of P19,286.79. 1. The fact that no separate premium was paid on the Cover Note
before the loss occurred does not militate against the validity of
the contention even if no such premium was paid. All Cover Notes do
not contain particulars of the shipment that would serve as basis for
INSURANCE 43
the computation of the premiums. Also, no separate premiums are Facts:
required to be paid on a Cover Note.

> On March 13, 1963, Pacific secured temporary insurance from the
The petitioner paid in full all the premiums, hence there was no
account unpaid on the insurance coverage and the cover note. If the
Workemens Insurance Co. for its exportation of logs to Japan.
note is to be treated as a separate policy instead of integrating it to Workmen issued on said date Cover Note 1010 insuring said cargo.
the regular policies, the purpose of the note would be meaningless.
It is a contract, not a mere application for insurance. > The regular marine policies were issued by the company in favor of
Pacific on Apr 2, 1963. The 2 marine policies bore the number
It may be true that the marine insurance policies issued were for 53H01032 and 53H01033.
logs no longer including those which had been lost during loading
operations. This had to be so because the risk insured against is for
loss during transit, because the logs were safely placed aboard. > After the issuance of the cover note but BEFORE the issuance of the
2 policies, some of the logs intended to be exported were lost due to a
The non-payment of premium on the Cover Note is, therefore, no typhoon.
cause for the petitioner to lose what is due it as if there had been
payment of premium, for non-payment by it was > Pacific filed its claim with the company, but the latter refused,
not chargeable against its fault. Had all the logs been lost during the
loading operations, but after the issuance of the Cover Note, liability
contending that said loss may not be considered as covered under the
on the note would have already arisen even before payment of cover note because such became null and void by virtue of the issuance
premium. Otherwise, the note would serve no practical purpose in of the marine policies.
the realm of commerce, and is supported by the doctrine that where
a policy is delivered without requiring payment of the premium, the
presumption is that a credit was intended and policy is valid.
Issue:

2. The defense of delay cant be sustained. The facts show that Whether or not the cover not was without consideration, thus null and
instead of invoking the ground of delay in objecting to petitioner's void.
claim of recovery on the cover note, the insurer never had this in its
mind. It has a duty to inquire when the loss took place, so that it
could determine whether delay would be a valid ground of objection.
Held:

There was enough time for insurer to determine if petitioner was It was with consideration.
guilty of delay in communicating the loss to respondent company. It
never did in the Insurance Commission. Waiver can be raised against SC upheld Pacifics contention that said cover not was with
it under Section 84 of the Insurance Act. consideration. The fact that no separate premium was paid on the cover
note before the loss was insured against occurred does not militate
Pacific Timber v. CA against the validity of Pacifics contention, for no such premium could
have been paid, since by the nature of the cover note, it did not contain,
as all cover notes do not contain, particulars of the shipment that would
112 SCRA 199 serve as basis for the computation of the premiums. As a logical

INSURANCE 44
consequence, no separate premiums are required to be paid on a cover WIC then asked an adjuster to investigate the loss. The adjuster
note. submitted that the logs lost were not covered by the two policies issued
on April 2, 1963 but said logs were included in the cover note earlier
If the note is to be treated as a separate policy instead of integrating it to issued.
the regular policies subsequently issued, its purpose would be
meaningless for it is in a real sense a contract, not a mere application. WIC however denied the insurance claim of PTEC as it averred that the
cover note became null and void when the two policies were
subsequently issued. The Court of Appeals ruled that the cover note is
void for lack of valuable consideration as it appeared that no premium
payment therefor was made by PTEC.

ISSUE: Whether or not a separate premium is needed for cover notes.

HELD: No. The Cover Note was not without consideration for which
the Court of Appeals held the Cover Note as null and void, and denied
Pacific Timber Export recovery therefrom. The fact that no separate premium was paid on the
Cover Note before the loss insured against occurred, does not militate
Corporation vs Court of against the validity of PTECs contention, for no such premium could
have been paid, since by the nature of the Cover Note, it did not contain,
Appeals as all Cover Notes do not contain particulars of the shipment that would
serve as basis for the computation of the premiums. As a logical
consequence, no separate premiums are intended or required to be paid
on a Cover Note.
112 SCRA 199 Mercantile Law Insurance Law The Policy
Separate Premiums Not Required for Cover Notes At any rate, it is not disputed that PTEC paid in full all the premiums as
called for by the statement issued by WIC after the issuance of the two
In 1963, Pacific Timber Export Corporation (PTEC) applied for a regular marine insurance policies, thereby leaving no account unpaid by
temporary marine insurance from Workmens Insurance Company PTEC due on the insurance coverage, which must be deemed to include
(WIC) in order for the latter to insure 1,250,000 board feet of logs to be the Cover Note. If the Note is to be treated as a separate policy instead
exported to Japan. In March 1963, WIC issued a cover note to PTEC for of integrating it to the regular policies subsequently issued, the purpose
the said logs. On April 2, 1963, WIC issued two policies for the logs. and function of the Cover Note would be set at naught or rendered
However, the total board feet covered this time is only 1,195,498. On meaningless, for it is in a real sense a contract, not a mere application
April 4, 1963, while the logs were in transit to Japan, bad weather for insurance which is a mere offer.
prevailed and this caused the loss of 32 pieces of logs.

INSURANCE 45

You might also like